Emergency Medicine Core Competency

Réussis tes devoirs et examens dès maintenant avec Quizwiz!

Question: Evaluation of extraocular movement on physical exam in patients with facial trauma is utilized to rule out what complication of orbital trauma?

Answer: Orbital fracture with associated muscle entrapment.

Question: Cat bite wounds are classically contaminated with what organism?

Answer: Pasteurella multocida.

Question: What is the largest sesamoid bone in the body?

Answer: Patella.

Question: What is the most common major neurologic complication of knee dislocations?

Answer: Peroneal nerve injury.

Question: Which medication is used to treat atrial fibrillation with Wolff-Parkinson-White syndrome?

Answer: Procainamide.

Question: Which two factors determine the risk of fetal death due to abruptio placentae?

Answer: Rapidity of separation and degree of placental surface affected.

Question: An impaired Achilles tendon reflex is associated with damage to which spinal root?

Answer: S1.

Question: What are two sites to perform needle decompression for tension pneumothorax?

Answer: Second intercostal space in the midclavicular line, and fourth or fifth intercostal space in the anterior axillary line.

Question: How is a positive likelihood ratio calculated?

Answer: Sensitivity divided by (1 - specificity).

Question: What is the most common acute complication of a nasal septal hematoma?

Answer: Septal abscess.

Question: What medical therapy is recommended to lessen the severity and duration of pain associated with a peritonsillar abscess?

Answer: Single use of high-dose intravenous corticosteroid (e.g., methylprednisolone).

Question: What is the most common location for the conceptus to implant in an ectopic pregnancy?

Answer: Some 95% are in the fallopian tubes.

Question: Which organ is most commonly injured in blunt abdominal trauma?

Answer: Spleen.

Question: What is Seidel sign?

Answer: Streaming fluorescent dye indicating leakage of aqueous humor due to a corneal or scleral laceration.

Question: What is the radiographic finding that suggests epiglottitis on lateral neck radiograph?

Answer: The "thumb sign".

Question: What exam finding is pathognomonic for torsion of a testicular appendage?

Answer: The blue dot sign - a blue discoloration on the upper pole of the testis.

Question: What body part is most commonly affected by high pressure injection injuries?

Answer: The index finger of the nondominant hand is most commonly affected.

Question: What determines the degree of cardiovascular instability from a pericardial effusion?

Answer: The rate of fluid accumulation (not the absolute volume).

Question: What bone is most commonly involved in basilar skull fractures?

Answer: The temporal bone.

Question: Are most abdominal aortic aneurysms located above or below the renal arteries?

Answer: The vast majority of aortic aneurysms are infrarenal in location.

Question: What are the general treatment guidelines for fifth metatarsal stress fractures or fractures that are distal to the intermetatarsal joint?

Answer: These do not typically heal with conservative management and may require surgical pinning by orthopedics.

Question: What patient characteristics are more likely in those presenting with a de Winter ECG pattern?

Answer: These patients are typically younger, more often male, and more likely to have hypercholesterolemia compared with those who presented with classic ST elevation.

Question: What is the most common injury leading to compartment syndrome?

Answer: Tibial fracture.

Question: True or false: Pencil graphite and glass are visible on plain films.

Answer: True.

Question: A slow progressive onset and associated prodrome is consistent with which type of syncope?

Answer: Vasovagal.

Question: What are the three patterns of pelvic ring fractures in the Young-Burgess classification?

Answer: Vertical shear, lateral compression and anterior-posterior compression

Question: Which needle causes fewer post-LP headaches: Whitacre (pencil tip with a side notch) or Quincke (bevel-type needle)?

Answer: Whitacre.

Question: Can the "halo sign" differentiate CSF from other body fluids?

Answer: It is not specific to CSF and can be seen with other fluids such as saliva or tears.

Question: What nondepolarizing agents can be considered if there are contraindications to succinylcholine?

Answer: Pancuronium, rocuronium and vecuronium.

Question: What patient population is more likely to have quadriceps tendon injuries compared to patellar tendon injuries?

Answer: Patients over the age of 40.

Question: In what subset of patients is atropine not effective?

Answer: Patients with a history of cardiac transplant.

Question: What are the five recommended sites for intraosseous placement?

Answer: Proximal tibia, distal tibia, distal femur, proximal humerus and sternum.

Question: What other injury often accompanies flail chest?

Answer: Pulmonary contusion.

Question: What is Kehr's sign?

Answer: Referred left shoulder pain due to subdiaphragmatic irritation or splenic rupture.

Question: Which chamber is most commonly injured in penetrating cardiac trauma?

Answer: Right ventricle.

Question: Which organism should be covered for septic arthritis in patients with sickle cell disease?

Answer: Salmonella.

Question: Is the FAST exam more sensitive for detecting solid organ injury or intraperitoneal fluid?

Answer: The FAST exam is highly sensitive and specific for the detection of free intraperitoneal fluid. It has poor sensitivity for solid-organ injury, hollow viscous injury, and retroperitoneal injury.

Question: What is the expected ultrasound appearance of the IVC in the setting of pericardial tamponade?

Answer: The IVC will appear plethoric (dilated and noncollapsible with respiration).

Question: What is the most superficial fascial layer of the neck?

Answer: The investing fascia.

Question: What portion of the temporal bone is most likely to be involved in basilar skull fractures?

Answer: The petrous portion.

Question: What is the most commonly injured organ in adult blunt abdominal trauma? In children?

Answer: The spleen is the most commonly injured organ in adults and children.

Question: Is retinal detachment painful?

Answer: The tear itself is not painful. Therefore, it is often classified as a painless loss of vision.

Question: What is a quantitative test that assesses the degree of materno-fetal hemorrhage?

Answer: Kleihauer-Betke test.

Question: What three signs are used in the evaluation of pneumothorax with ultrasound?

Answer: Lack of lung sliding, bar code sign on "M" mode, presence of a transition point.

Question: What two traumatic mechanisms in pediatric patients frequently cause a duodenal hematoma?

Answer: Lap belt injuries and handlebar injuries.

Question: What is the emergency department treatment for orbital compartment syndrome

Answer: Lateral canthotomy.

Question: Which LeFort fracture carries an increased risk of cerebrovascular involvement?

Answer: LeFort II and III.

Question: What is the normal diameter for the optic nerve sheath?

Answer: Less than 5 mm.

Question: What is the most prevalent type of malpractice claim made in the U.S. against emergency department providers?

Answer: Missed or delayed diagnosis.

Question: What procedure can be utilized to aid in identifying the aorta during a thoracotomy?

Answer: Nasogastric tube placement can help differentiate the esophagus from the aorta.

Question: What is the most common cause of cardiac arrest in pediatric patients?

Answer: Hypoxia.

Question: What is the mechanism of action for norepinephrine?

Answer: Norepinephrine functions mainly as a peripheral vasoconstrictor by acting on alpha-adrenergic receptors.

Question: When performing IVC ultrasound, the IVC should be visualized entering what structure?

Answer: IVC should be seen draining into the right atrium to definitively differentiate it from the aorta.

Question: Which has been shown to be more effective in abscess management: incision and drainage or ultrasound-guided aspiration?

Answer: Incision and drainage.

Question: What technique may improve the chance of diagnosing a Lisfranc injury on plain radiograph?

Answer: Including weight-bearing (stress) views.

Question: Where is the most common location for aortic aneurysms to occur?

Answer: Infrarenal; the aorta at this location is tapered in diameter relative to the proximal portion leaving the infrarenal segment with the most wall tension.

five-letter coding scheme was developed in the 1970s as a way to standardize pacemaker nomenclature.

A five-letter coding scheme was developed in the 1970s as a way to standardize pacemaker nomenclature. The first letter represents the chamber that is paced and can be a V (ventricle), A (atrium), D (dual chamber) or O (none). The second letter, also a V, A, D, or O, represents the chamber that is sensed. The third letter represents the sensing response and can be a T (triggered), I (inhibited), D (dual mode of response) or O (none). The fourth letter refers to the programmability of the pacemaker and can be simple (P), multiprogrammable (M), rate adaptive (R), communicating (C) or none (O). The fifth letter, which is rarely used, represents the presence or absence of multisite pacing. As an example, a VVI pacemaker will pace only the ventricle, senses intrinsic depolarization of the ventricle and will inhibit the output pulse based on a sensed event.

Question: What is the standard initial bilevel positive airway pressure setting?

Answer: Inspiratory positive airway pressure of 10 cm H2O and a positive end expiratory pressure of 5 cm H2O.

Question: What is the first-line treatment for Lyme arthritis?

Answer: Doxycycline 100 mg orally twice a day for four weeks.

Question: What pacemaker malfunction occurs when the generator fires but the current delivered to the endocardium is not sufficient to initiate depolarization?

Answer: Failure to capture.

Question: True or false: In cases of respiratory depression, changes in oxygen saturation will occur before changes in end tidal CO.

Answer: False. Changes in end tidal CO2 are noted well before a patient is noted to have changes in oxygen saturation.

Question: What condition can unmask ECG abnormalities in a patient with Brugada syndrome?

Answer: Fever.

Question: A flat anterior chamber indicates what diagnosis?

Answer: Globe rupture.

Question: In what traumatic ocular injury should intraocular pressure never be measured?

Answer: Globe rupture.

Question: What are risk factors for non-traumatic uterine rupture?

Answer: Greater than three prior cesarean deliveries and prior vertical uterine incision.

Question: What should an avulsed tooth be placed in?

Answer: Hank solution or milk.

Question: What dose of ketamine has been most commonly used to treat patients with chronic pain in the emergency department?

Answer: 0.3 mg/kg infused over 10 minutes.

Question: What is the urine output goal during resuscitation of burns?

Answer: 1 mL/kg/hour.

Question: What is a patient's GCS if they are opening their eyes spontaneously, confused and able to localize pain?

Answer: 13.

Question: What is the upper limit of normal measurement of the diameter of the abdominal aorta?

Answer: 3 cm.

Question: What is the upper limit of normal of the gallbladder wall?

Answer: 3 mm.

Question: What is the sensitivity of detecting retinal detachment by ocular ultrasound?

Answer: 97-100% when performed by an emergency physician.

Question: What is the emergency severity index (ESI)?

Answer: A 5-point scale used to estimate patient acuity and anticipated resource needs at triage. ESI 1 corresponds to an immediate life-threatening circumstance; ESI 2 indicates a high-risk situation as defined by vital signs, severe pain, or distress.

Question: What is an occult pneumothorax?

Answer: A pneumothorax that is absent on chest X-ray but identified on subsequent computed tomography.

Question: What is the SAVE triage system?

Answer: A system designed to identify patients most likely to benefit from care under austere field conditions or in a resource-poor environment.

Question: What is a Barton's bandage?

Answer: Ace wrap over the top of the head and under the mandible use to stabilize mandibular fractures.

Question: Patients who do not comply with early passive mobilization of the shoulder after clavicle fractures may sustain what complication?

Answer: Adhesive capsulitis (frozen shoulder).

Question: If antibiotics for nasal fracture with anterior packing are prescribed, what would be the first line antibiotic choice?

Answer: Amoxicillin-clavulanate.

Question: Name a medication that is considered as one of the first line treatment to prevent cardiovascular diseases?

Answer: Aspirin (number needed to treat approximately 44).

Question: What beta-blocker is preferred for asthmatic patients?

Answer: Beta-1 selective drugs such as atenolol, esmolol or metoprolol.

Question: What are the preferred agents for blood pressure control in patients with traumatic aortic injuries?

Answer: Beta-blockers (if not contraindicated).

Question: What diagnosis should be considered in patients with urinary retention, back pain, hyporeflexia, and perineal sensory deficits?

Answer: Cauda equina syndrome.

Question: What are the borders of Zone I of the neck?

Answer: Clavicles to the cricoid cartilage.

Question: Acids cause what type of tissue damage?

Answer: Coagulation necrosis.

Question: What is the mortality rate of patients with sternal fractures?

Answer: Contrary to prior belief, sternal fractures are associated with < 1% mortality.

Question: Aside from supportive care and benzodiazepines, which drug can be used to treat severe serotonin syndrome?

Answer: Cyproheptadine.

Question: What is the most common location of Salter-Harris type I fractures?

Answer: Distal fibula.

Which if the following organs is most commonly injured in pediatric blunt abdominal trauma? Liver Pancreas Spleen Stomach

Correct Answer ( C ) Explanation: Blunt trauma is the most common mechanism of abdominal trauma in pediatric patients. Splenic injuries are the most common injury in pediatric abdominal trauma. The abdomen begins at the level of the nipple in pediatric patients. The pliable cartilaginous rib cage increases the risk of compression of abdominal organs. This leads to a high risk of significant intra-abdominal injuries with few external signs of trauma. The risk for splenic injury is high as children have larger solid organs, less protective muscle, and less subcutaneous fat. Pelvic injuries are much less common in children than adults, and pelvic fractures in pediatric patients rarely lead to life-threatening hemorrhage the way they do in adult patients. The liver (A) is the second most commonly injured large organ in children with blunt traumatic injuries. The pancreas (B) may be injured in blunt handlebar injuries to the epigastrium in pediatric patients. The stomach (D) is rarely injured in blunt abdominal trauma in pediatric patients.

19-year-old woman with a past medical history of fibromyalgia and chronic back pain presents to the Emergency Department with fever, headache and altered mental status. She was involved in a motor vehicle collision two days prior. Serum and urine studies are sent along with blood cultures. A noncontrast head CT is obtained and is normal. Which of the following would preclude performing a lumbar puncture at this time? Lumbar vertebral fracture Previous spinal surgery Scoliosis Spinal metastasis

Correct Answer ( A ) Explanation: A lumbar puncture should be performed on patients suspected of having meningitis or other infectious causes of headache or altered mental status. However, the benefits of the procedure must be weighed against the risks and any contraindications. Lumbar puncture is contraindicated in patients with vertebral trauma (e.g., lumbar vertebral fracture), overlying skin infection, CNS mass causing increased intracranial pressure, spinal mass (excluding metastatic disease), platelet count less than 20,000 cells/mm3, or coagulopathy (e.g., von Willebrand's disease, hemophilia, recent administration of heparin). If the lumbar puncture cannot be performed and meningitis is suspected, antibiotics should be administered empirically. Imaging with a noncontrast head CT prior to lumbar puncture is indicated for patients with an altered sensorium, focal neurologic deficits, papilledema, new-onset seizures and immunosuppression (e.g., chronic steroid use, history of cancer or human immunodeficiency syndrome). Previous spinal surgery (B) and scoliosis (C) are not contraindications for an lumbar puncture. However, these features can make the procedure more difficult. Fluoroscopic guidance may be necessary to reduce the number of operator attempts and patient discomfort, and maximize success of the procedure. Spinal metastasis (D) is not currently a contraindication for performing a lumbar puncture.

An 84-year-old woman presents to the emergency department by ambulance after falling at home two hours prior to arrival. She is complaining of a headache. She has a history of atrial fibrillation and is on warfarin. On physical exam she has a large temporal scalp hematoma and her GCS is 12. Which of the following mechanisms led to the pathology seen on the image above? Injury to the bridging veins Injury to the middle meningeal artery Ruptured berry aneurysm Ruptured microaneurysms

Correct Answer ( A ) Explanation: A subdural hematoma (SDH) results from injury to the bridging veins in the subdural space. Brain atrophy increases the risk of this injury and it is more frequently seen in the elderly and alcoholics. Acute SDHs typically become symptomatic within 24 hours following trauma with headache, vomiting or altered mental status. On CT, the appearance of blood is a concave, crescent shape. Subdural hematomas may cross suture lines, but they do not cross the midline. Acute subdural hematomas with a thickness greater than 10 mm or a midline shift greater than 5 mm on CT should be surgically evacuated. Injury to the middle meningeal artery (B) results in an epidural hematoma. On CT the appearance of blood is a convex, lenticular shape. Epidural hematomas may cross the midline, but do not cross suture lines. A ruptured berry aneurysm (C) results in a hemorrhagic stroke in the form of a subarachnoid hemorrhage. Ruptured microaneurysms (D) result in a hemorrhagic stroke in the form of an intracerebral hemorrhage, most often in the basal ganglia.

Which of the following is a required element in malpractice allegations claiming physician negligence? Causal connection between negligence and alleged damages Death or permanent injury to the patient Failure to appropriately diagnose a patient's condition Failure to disclose adverse effects of treatment

Correct Answer ( A ) Explanation: All of the elements of negligence in a malpractice allegation must be satisfied in order for a provider to be found guilty of malpractice for negligence. These 4 elements are: Actions that would not be reasonably expected of a provider cannot lead to a finding of negligence. Failure to diagnose (C) is neither necessary for a negligence claim nor sufficient to qualify as negligent. Although some injury must occur to the patient as a result of the alleged negligence, it need not result in death or permanent injury (B). Failure to disclose adverse effects (D) of a treatment is not required for a claim of negligence, although it does represent unethical behavior.

A 20-year-old man presents to the Emergency Department with acute onset left testicular pain that woke him from sleep. He denies any recent trauma or surgery. He has not been sexually active in over one year. He has an absent cremasteric reflex on the left with an elevated left testis in horizontal lie. Which of the following is the most appropriate next step in management in the absence of urgent urologic consultation? Lateral testicular rotation Oxycodone-acetaminophen Ultrasound Urinalysis and urine culture

Correct Answer ( A ) Explanation: An acutely painful scrotum may be caused by testicular torsion, torsion of the testicular appendage, epididymitis, trauma, or an incarcerated hernia. Testicular torsion is a urologic emergency as the spermatic cord and vasculature to the testis may become compromised leading to testicular necrosis and infertility. The majority of cases occur in the absence of trauma or other inciting event and may occur during sleep. Patients may complain of testicular pain, lower abdominal pain or pain in the inguinal canal. A testis oriented in a horizontal or transverse lie is particularly at risk for being torsed. On physical examination, the affected hemiscrotum appears enlarged, elevated compared to the unaffected side, and is exquisitely tender to touch or manipulation. An absent unilateral cremasteric reflex is highly sensitive (but less specific) for torsion. Emergent urologic consultation is needed in addition to obtaining a doppler ultrasound of the scrotum. However, prior to imaging or consultation, manual detorsion should be attempted for presumed torsion. Intravenous analgesia may be required for successful manipulation. Manual detorsion should be performed with lateral testicular rotation as if opening a book. The procedure itself is painful, but patients should be counselled that this may relieve pain from torsion and improve testicular survival if successful. Oxycodone-acetaminophen (B) is not appropriate because the patient may require operative intervention and should receive nothing by mouth until surgical options are discussed. Intravenous analgesia is indicated however. Ultrasound (C) would delay definitive management. Early surgical consultation is needed, and imaging should not delay definitive care. Urinalysis and urine culture (D) should be sent to assess for infectious processes such as prostatitis, epididymitis, uCorrect Answer ( A )

71-year-old man with a history of hypertension and diabetes mellitus presents with a painful left knee. His vital signs are T 102.6º F, HR 107 beats/minute, BP 130/90 mm Hg, RR 12 breaths/minute. His left knee appears swollen and feels warm compared to the right knee. There is no overlying erythema. He has significant pain with attempted active and passive range of motion of the left knee. In addition to analgesia, which of the following is the next most appropriate step in management? Arthrocentesis Knee radiography Orthopedic consultation Prednisone

Correct Answer ( A ) Explanation: Any patient with a newly painful, swollen joint with restricted range of motion and fever has septic arthritis until proven otherwise. Patients usually complain of joint pain, swelling, and fever. Rigors and chills are less common. Arthrocentesis is critical to making the diagnosis, but no single laboratory value is diagnostic of septic arthritis. Synovial fluid analysis should be performed to assess for bacterial growth. A synovial white blood cell count greater than 50,000 cells/mm3 has a sensitivity of approximately 56% for nongonococcal septic arthritis. Antibiotics covering Staphylococcal and Streptococcal species, including methicillin-resistant Staphylococcus aureus, should be administered. Recommended therapy includes vancomycin and a third-generation cephalosporin. If synovial fluid analysis is equivocal and the diagnosis cannot be excluded, admission and orthopedic consultation is recommended as operative joint irrigation may be indicated. Knee radiography (B) adds little in making the diagnosis of septic arthritis. In the absence of history or exam findings suggesting acute trauma or foreign body, X-rays are not indicated. Orthopedic consultation (C) should occur after arthrocentesis has been performed and synovial fluid studies have been sent. Additionally, if overlying cellulitis precludes arthrocentesis and clinical suspicion for septic arthritis is high, early consultation is appropriate. However, given the broad differential diagnosis for a warm, swollen and painful joint (e.g., gout, pseudogout, septic and nonseptic bursitis), consultation should be made judiciously. Prednisone (D) is not appropriate therapy for septic arthritis. For non-diabetic patients with chronic kidney disease, this is considered a first-line treatment for gout. Given this patient's fever and tachycardia, septic arthritis is the more likely diagnosis.

What diagnosis is demonstrated on this transabdominal ultrasound? Abdominal aortic aneurysm Cholecystitis Free pelvic fluid Renal cyst

Correct Answer ( C ) Explanation: Ultrasound interpretation begins with identification of key landmarks. One key landmark shown here is the urinary bladder, identifiable by the appearance of anechoic (black) fluid (e.g., urine) within a structure defined by hyperechoic (white) appearing borders. Any anechoic collection outside the urinary bladder represents free fluid. Free fluid in the pelvis may be physiologic or pathologic (e.g., blood, ascites, contents from a ruptured ovarian cyst) in origin. Its presence should be considered within the respective clinical context. The abdominal aorta (A) bifurcates into the common iliac arteries proximal to the pelvic inlet. A thickened gallbladder wall, pericholecystic fluid, and gallstones characterize cholecystitis (B). Renal cysts (D) are smooth, round, anechoic structures often encountered as an incidental finding on transabdominal ultrasound.

18-year-old man is brought in by EMS following a motor vehicle collision with prolonged extrication. His vitals on arrival are HR 110, BP 100/60, RR 20. He is noted to have periorbital ecchymosis with tarsal plate sparing on physical examination. Which of the following is the most likely diagnosis? Basilar skull fracture Globe rupture Orbital floor fracture Subdural hematoma

Correct Answer ( A ) Explanation: Basilar skull fractures are those which involve any of the five bones comprising the skull base, including the cribriform plate, the occipital bones, the sphenoid bones, the squamous and petrous portions of the temporal bones, and the orbital plate of the frontal bones. Basilar skull fractures are often associated with extra-axial hematomas, especially epidural hematomas due to the involvement of the middle meningeal artery. Cerebrospinal fluid (CSF) leaks are also possible due to dural tears. Physical examination findings include mastoid ecchymosis (Battle sign), hemotympanum, CSF leaks (presenting as otorrhea or rhinorrhea), and periorbital ecchymosis with tarsal plate sparing ("raccoon eyes"). Basilar skull fractures may be suspected clinically but diagnosis needs to be confirmed with imaging. Any patient with a suspected basilar skull fracture should undergo a head CT to look for extra-axial hematoma formation as well as cerebral damage, which is common. Management of basilar skull fractures is dependent on presentation and involved structures. Extra-axial hematomas associated with a basilar skull fracture are a neurosurgical emergency. CSF leaks in the setting of basilar skull fractures are typically self limited; meningitis prophylaxis in this setting is controversial.

34-year-old man is involved in a motorcycle collision and presents to the Emergency Department with paramedics. His vital signs are T 98.9ºF, BP 90/65 mm Hg, HR 110 beats/minute, RR 30 breaths/minute, oxygen saturation 91% on room air. An upright chest X-ray is shown above. Which of the following is the most likely diagnosis? Diaphragm rupture Pneumoperitoneum Pulmonary contusion Tension pneumothorax

Correct Answer ( A ) Explanation: Diaphragm rupture can occur from blunt abdominal trauma (most common) or penetrating abdominal trauma and is almost always left-sided. The right hemidiaphragm is protected by the liver. In cases of blunt trauma, the intra-abdominal pressure becomes abruptly greater than the intrathoracic pressure, causing diaphragmatic tearing and herniation of abdominal contents through the diaphragm defect. A plain radiograph of the chest is the initial imaging modality of choice for diagnosis and may show loops of bowel in the hemithorax. If a nasogastric or orogastric tube is in place and the stomach has herniated through the hemidiaphragm defect, the tube will appear coiled in the thoracic cavity as opposed to in the expected abdominal location. However, plain radiography may not demonstrate injury if performed early after the injury. Computed tomography may better demonstrate a diaphragmatic injury, but the exact location and type of injury is often indeterminate. This is a critical diagnosis that should be made promptly as delay in treatment may lead to abdominal content herniation or strangulation, or compressive respiratory compromise. Emergent surgical repair is indicated. Pneumoperitoneum (B) is free air in the abdominal cavity due to hollow viscus rupture or perforation. It may be iatrogenic (e.g., endoscopy), spontaneous (e.g., erosive peptic ulcer) or traumatic. Extraluminal air appears as radiolucency underneath the diaphragm on an upright chest X-ray. Radiographic findings of pulmonary contusion (C) may not appear until 12 to 24 hours after the insult or injury. Patchy ground-glass opacities or diffuse consolidation may be seen. Tension pneumothorax (D) should be diagnosed clinically before imaging is obtained. Classic clinical findings include hypotension, tracheal deviation away from the pneumothorax, distended neck veins, and diminished breath sounds on the side of the pneumothorax. If imaging is obtained, the affected side will shows a pneumothorax with mass effect of mediastinal structures away from from the pneumothorax.

Correct Answer ( A ) Explanation: Inferior vena cava ultrasound can be used to estimate the patient's central venous pressure and can help determine if hypotension is likely due to a low preload or elevated afterload. A caval index > 50% suggests fluid responsiveness. The caval index is calculated as (IVC expiratory diameter - IVC inspiratory diameter) / (IVC expiratory diameter) * 100. In this case, the caval index is calculated as (1.6 - 0.7) / 1.6 * 100 = 56% which indicates the patient has low preload and suggests fluid responsiveness in his blood pressure with IV fluid hydration.

Correct Answer ( A ) Explanation: Inferior vena cava ultrasound can be used to estimate the patient's central venous pressure and can help determine if hypotension is likely due to a low preload or elevated afterload. A caval index > 50% suggests fluid responsiveness. The caval index is calculated as (IVC expiratory diameter - IVC inspiratory diameter) / (IVC expiratory diameter) * 100. In this case, the caval index is calculated as (1.6 - 0.7) / 1.6 * 100 = 56% which indicates the patient has low preload and suggests fluid responsiveness in his blood pressure with IV fluid hydration. Initiation of a dobutamine drip and administering IV furosemide (B) would be treatments used for an acute congestive heart failure (CHF) exacerbation. If this patient was hypotensive from CHF, one would expect to see an elevated IVC diameter (>2.5 cm) and decreased respiratory variation (< 50% collapse with inspiration) as hypotension, in this case, is due to increased afterload. Dobutamine must be used cautiously in hypotensive patients as its beta-adrenergic effects can cause peripheral vasodilation and worsening hypotension. Initiation of an esmolol drip and obtaining vascular surgery consultation (C) would be the management strategy for an aortic dissection. The aorta is often confused with the IVC on ultrasound. However, the aorta is located to the left of the IVC, does not collapse, has pulsatile flow, and has a thicker, multilayered wall. Initiation of a heparin drip and obtaining a CT pulmonary angiogram (D) would be the management strategy for a suspected pulmonary embolism (PE). Hypotension caused by a PE is due to obstructive shock and would cause a dilated IVC without respiratory variation, similar to an acute CHF exacerbation.

What can be done to help alleviate the most common complication of intraosseous line placement? Administer 2% lidocaine without epinephrine through the intraosseous line Administer cefazolin immediately after resuscitation through the intraosseous line Administer subcutaneous epinephrine near the intraosseous insertion site Obtain a radiograph before use of the intraosseous line

Correct Answer ( A ) Explanation: Intraosseous access in an excellent adjunctive device, particularly in small children, for allowing access to the central circulation. It is indicated when there is an emergent need for vascular access when other sites are difficult or time-consuming to establish. Any medication or fluid that can be infused through an intravenous line can be administered through an intraosseous line but there may be a slightly delayed effect. The main limitation of intraosseous access is a relatively low flow rate due to resistance in the bone marrow cavity. However, this resistance can be overcome with pressure bags. There are few complications from intraosseous placement and the rate of serious complications is less than the rate of serious complications from central line access. The most common complication is pain during insertion and infusion. This pain can be alleviated by administration of 2% lidocaine without epinephrine at a dose of 0.5 mg/kg, followed by a normal saline flush.

Which of the following is a hard sign for aerodigestive injury in the setting of penetrating neck trauma? Bubbling wound Dysphagia Hoarse voice Subcutaneous emphysema

Correct Answer ( A ) Explanation: Penetrating neck injuries are potentially emergent injuries due to the numerous aerodigestive, vascular, and neurological structures that run through the neck. A penetrating neck injury is defined by penetration of the platysma, a thin muscle stretching from the facial muscles to the thorax that demarcates superficial from deep injuries of the neck. Patients with penetrating neck injuries should be evaluated for hard signs of aerodigestive or vascular injury, which include shock unresponsive to fluids, active arterial bleeding, pulse deficit, massive hemoptysis, pulsatile or expanding hematoma, vascular thrill or bruit, stridor or airway obstruction, cerebral ischemia, or bubbling in the wound. Soft signs include a non-expanding hematoma, proximity wounds, hoarseness, neck tenderness, subcutaneous emphysema, dysphagia, saliva draining from the wound, or prevertebral air. Nine out of ten patients with hard signs of aerodigestive or vascular injury will have an injury requiring surgical repair and these patients should be immediately transferred to the operating room. Dysphagia (B), voice changes including hoarse voice (C), and subcutaneous emphysema (D) are all soft signs of aerodigestive and neurovascular injury in the setting of neck trauma. Patients with soft signs should raise suspicion for structural neck injury and warrant further diagnostic testing by angiography, bronchoscopy, or endoscopy. Only a small percentage of patients with soft signs of aerodigestive or vascular injury will have a clinically significant injury

77-year-old woman presents to the emergency department with weakness. She has a history of hypothyroidism and is on levothyroxine. On physical examination her blood pressure is 90/60 mm Hg, heart rate 38 bpm, temperature 98.9°F, respirations 18 bpm. She is oriented only to herself and is notably confused. An ECG reveals sinus bradycardia. An IV is established. The most appropriate next step is administration of which of the following? Atropine Dopamine Epinephrine Transvenous pacing

Correct Answer ( A ) Explanation: Sinus bradycardia on ECG appears as a regular rhythm with ventricular rates less than 60 beats per minute. The P wave morphology and PR intervals are normal and consistent. If the patient is unstable with either hypotension, altered mental status, or signs of shock, then intervention is necessary. Initial interventions include atropine and transcutaneous pacing. Atropine is a nonselective muscarinic acetylcholinergic antagonist. Its cardiac effects include increasing the firing of the sinoatrial (SA) node and the conduction through the atrioventricular (AV) node. The initial dose is 0.5 mg IV/IO. It can be repeated every 3-5 minutes up to a maximum of 3 mg. Transcutaneous pacing should be considered immediately in any unstable patient, especially those with high-grade AV blocks. Beta-agonists, such as dopamine (B) and epinephrine (C), are considered second-line agents when atropine has failed. Transvenous pacing (D) is the definitive treatment for persistent and symptomatic bradycardia.

22-year old man presents to the ED with an isolated stab wound to the chest. His BP is 80/40, pulse 120, respiratory rate 32 and oxygen saturation 93%. Which of the following is the most likely cause of his hypotension based on the following FAST image? Cardiac tamponade Cardiogenic shock Hemorrhagic shock Septic shock

Correct Answer ( A ) Explanation: The FAST image shows a pericardial effusion. Given the clinical scenario of a stab wound to the chest and hypotension along with these ultrasound findings, cardiac tamponade is the likely cause of his hypotension. Cardiac tamponade occurs when blood accumulates in the pericardium and causes elevated intrapericardial pressure. This causes decreased right ventricular (RV) and left ventricular (LV) filling during diastole. If enough fluid accumulates, the myocardial septum will deviate and further decrease LV filling. This results in decreased cardiac output. The classic physical exam findings are hypotension, muffled heart sounds and jugular venous distention, which are referred to as Becks triad. Other physical exam findings include tachycardia, elevated central venous pressures and pulsus paradoxus. The patient should be resuscitated according to ATLS protocol, prepped for possible ED pericardiocentesis and ultimately will require definitive surgical repair by trauma a surgeon. Cardiogenic shock (B) is defined as decreased cardiac output despite adequate or even excessive circulating volume and is commonly seen in the setting of acute myocardial infarction. This is unlikely in the setting of penetrating trauma. Hemorrhagic shock (C) results from massive blood loss as a result of trauma. The clinical signs and symptoms are similar to that seen in cardiac tamponade. Although often seen in the setting of penetrating trauma, this is unlikely the sole cause of this patients hypotension given the ultrasound findings. Septic shock (D), as manifested by refractory hypotension in the setting of infection, SIRS and end organ dysfunction is unlikely to be the cause of hypotension in this clinical scenario.

35-year-old man is brought in by EMS after being the restrained driver in a high-speed vehicle collision. On examination, he is noted to have malocclusion. When the forehead is held stationary and the upper teeth are grasped, there is movement of the maxilla without any movement of the nasal bridge. Which of the following best classifies the likely fracture pattern in this patient? LeFort I LeFort II LeFort III LeFort IV

Correct Answer ( A ) Explanation: The LeFort fractures are midface fractures that involve either complete or partial detachment of the maxilla from the skull. They are classified by their location and the bones involved in the fracture pattern. LeFort I fractures involve a transverse fracture line through the maxilla, separating the maxilla from the pterygoid plate and the nasal septum and detaching the hard palate from the skull. On examination, patients will have movement of the maxilla when the upper teeth are rocked while the forehead is held steady. Other physical examination findings include swelling and ecchymosis of the face. Unlike in LeFort II and III fractures, LeFort I fractures are rarely associated with cerebrovascular involvement. Maxillofacial CT is the imaging modality of choice. Management is surgical. A LeFort II (B) fracture is a pyramidal fracture pattern that includes the nasal bridge, maxilla, lacrimal bones, orbital floor, and orbital rim. In this fracture pattern, the entire nasal complex will move when the maxilla is rocked with the forehead held steady. A LeFort III (C) fracture, also known as craniofacial dislocation, is characterized by a fracture line that travels transversely through the midface at the level of the nasal bridge and extends through the zygomatic arches. On examination, there is complete dissociation with motion of the entire midface from the cranium when the mandible is rocked while the forehead is held steady. The LeFort classification does not include a LeFort IV (D) fracture pattern.

27-year-old woman has pleuritic chest pain and mild shortness of breath. A D-dimer was performed and is negative. She questions the provider how sure she is that she does not have a pulmonary embolism given this negative test result. Which value will answer her question? Negative predictive value Positive predictive value Sensitivity Specificity

Correct Answer ( A ) Explanation: The negative predictive value is the likelihood that a person with a negative test does not have the disease in question. A positive predictive value (B) allows one to understand the likelihood of having a disease with a positive test. This is calculated by A/(A+B), or the number of patients with disease and a positive test result divided by the cohort of all patients with positive test results. Sensitivity (C) refers to the test's ability to correctly detect patients who do have the condition. The sensitivity of the test is the proportion of people who test positive for the disease among those who have the disease, A/(A+C). These tests are frequently used as helpful rule out tests because with a high sensitivity a negative test result effectively rules out disease because most people with the disease have a positive test. Specificity (D) relates to the test's ability to correctly detect patients without a condition. Specificity of a test is the proportion of healthy patients known not to have the disease, who will test negative for it, D/(B+D). These tests are more helpful if they are positive reflecting a higher chance of an individual having a disease.

45-year-old woman presents to the emergency department with pain all over her body. She reports that she has a chronic pain syndrome but she ran out of her medication one week ago. Since that time, her pain has become unbearable and is focused mainly in her neck, back and legs. Examination reveals tenderness over the paraspinal region of her thoracic and lumbar spine and over her trapezius muscles bilaterally. Otherwise, her examination is unrevealing. What class of medication is most likely to reduce this patient's pain? Antidepressant Antipsychotic Corticosteroid Opioid

Correct Answer ( A ) Explanation: The patient in this question is describing a chronic pain syndrome that is characteristic of fibromyalgia. Fibromyalgia is defined as widespread pain lasting greater than three months not attributed to another disorder and associated with a constellation of symptoms, including fatigue, stiffness, difficulty moving, cognitive dysfunction, tenderness to palpation, sleep disturbances, anxiety and depression. Three medications have been approved by the Federal Drug Administration for the treatment of fibromyalgia: duloxetine and milnacipran, antidepressants, and pregabalin, an anticonvulsant.Correct There is some discussion that quetiapine, an antipsychotic (B), appears to improve depression and reduce pain in patients with fibromyalgia but none of the other antipsychotics have shown this improvement and quetiapine is not recommended as first-line therapy. Corticosteroids (C) have been recommended for patients with complex regional pain syndrome, a disorder that typically begins four to six weeks following a minor or moderate extremity injury, characterized by severe chronic pain and disability affecting the limb. Corticosteroids have been recommended at the time of first presentation of patients with this syndrome when signs of inflammation are still present. However, steroids do not affect the course of illness. The patient in this question has not reported a recent injury. With the possible exception of tramadol, opioids (D) are ineffective for the treatment of fibromyalgia and are not recommended in chronic non-cancer pain patients. Additionally, use of opioids may lead to poorer patient outcomes.

26-year-old woman presents to the Emergency Department after accidentally washing her hands with industrial strength bleach. Which of the following characteristics is true regarding an alkali burn? Causes epithelial cell desiccation Forms tough, leathery eschars Generates lipid saponification Scavenges ions within tissues

Correct Answer ( C ) Explanation: Chemical burns can be classified into four categories: acids, alkalis, organic solvents, or inorganic solvents. An alkaline burn causes liquefactive necrosis and generates lipid saponification. This is a result of alkaline substances binding with proteins and lipids to form soluble protein complexes and soaps within tissues. These complexes then allow for deeper penetration of ions into tissues leading to soft, gelatinous eschars. They are more dangerous and lead to more systemic toxicity than other burns due to deeper penetration and longer duration of causticity. Burns may initially appear minor or superficial, but providers should have a high index of suspicion for more involved and ongoing tissue injury. Burns from acids cause epithelial cell desiccation (A) that can form tough, leathery eschars (B). Inorganic solvents scavenge ions within tissues (D) and then form salts

22-year-old man presents to the Emergency Department after being struck in the face during a fight in his college dorm. He has a nasal bridge deformity with a blue, grape-like mass protruding from the left nasal septum. The remainder of his exam is atraumatic. Which of the following is the most appropriate step in managing this patient? Bedside incision and evacuation Discharge home with otolaryngology follow up Emergency operative exploration Nasal packing and empiric antibiotics

Correct Answer ( A ) Explanation: This patient has a nasal septal hematoma which should be managed promptly with bedside incision and evacuation of the clot to prevent damage to the underlying cartilage. Careful inspection of the nose should be performed during the secondary trauma survey to assess for this, as it is easily missed if the index of suspicion is not high. It appears as a blue or purple mass against the nasal septum. It may be associated with other signs of facial trauma, including a nasal bridge deformity, periorbital ecchymosis, midface instability or facial lacerations. If a septal hematoma is suspected but not apparent on exam, inserting a gloved small finger and palpating along the septum may be indicated. There are no absolute contraindications to incising a septal hematoma. If left untreated, pressure necrosis of the nasal septum is likely to occur, resulting in collapse of the cartilaginous structures and a saddle nose deformity. Other complications include abscess formation and septal perforation. Septal hematomas should be incised and the hematoma evacuated completely. Needle aspiration is inadequate to fully remove the clot. If bilateral hematomas are present, both require incision and drainage with special care to avoid a through-and-through septal perforation. Broad-spectrum antibiotics should be administered prophylactically to cover Staphylococcus aureus, Haemophilus influenzae and Streptococcus pneumoniae. Amoxicillin-clavulanate is a widely accepted first-line treatment, and clindamycin is preferred if an abscess is present. Prompt otolaryngology follow up should be arranged.

A 35-year-old man presents to the ED after being assaulted in a bar fight. He has signs of significant facial trauma. On physical exam, you note a tooth fracture through the enamel and dentin but does not involve the pulp. Which of the following is the most appropriate next step in management of his dental injury? Calcium hydroxide paste Extract the tooth Immediate dental consult Penicillin VK

Correct Answer ( A ) Explanation: This patient has evidence of an Ellis II dental fracture. Fractures of the teeth are described by the Ellis classification system. Class I involves injury to the enamel alone. It is painless and is treated by smoothing any rough edges and dental referral. Class II involves injury to the dentin and enamel. There is often hot or cold sensitivity. Physical exam reveals exposed yellow dentin. Management is by covering the exposed dentin with calcium hydroxide paste. Patients should be urgently referred to an oral surgeon or dentist. Class III involves injury to the enamel, dentin, and pulp. Patients present with severe pain unless the neurovascular bundle is disrupted. There is pink-tinged pulp visible on exam. Management is with emergent oral surgery referral, preferably within 24 hours of the ED visit. Antibiotics should be given in these cases and penicillin VK is the antibiotic of choice. Extract the tooth (B) is not appropriate ED management of an Ellis II fracture. Immediate dental consult (C) and penicillin VK (D) are required for Ellis III fractures.

A 33-year-old woman presents to the ED with chest pain and dyspnea after being stepped on by a cow. On physical exam, you note paradoxical motion of part of her right chest wall. Breath sounds are present bilaterally. Which of the following is the most likely diagnosis? Flail chest Open pneumothorax Sequential rib fractures Tension pneumothorax

Correct Answer ( A ) Explanation: This patient is exhibiting signs and symptoms of flail chest. Flail chest occurs when a section of the ribs separates from the rest of the chest wall, usually via two fractures in each rib involving at least three adjacent ribs. Signs and symptoms of flail chest include chest pain and dyspnea. Palpation of the segment will usually reveal tenderness and bony crepitus. Inspection reveals paradoxical chest wall movement where the flail segment moves inward during inspiration and outward during expiration. Diagnosis is typically via physical examination and may be confirmed by chest radiograph. Computed tomography of the chest may be indicated if there is concern for concomitant chest or abdominal injuries. Treatment is via supportive care, which may include mechanical ventilation for respiratory failure and aggressive pulmonary toilet with adequate pain control. These patients should be admitted for further management. An open pneumothorax (B) is demonstrated by a sucking chest wound with diminished breath sounds on the ipsilateral side. This is managed by an occlusive dressing and a chest tube at a different site. Sequential rib fractures (C) will cause chest pain and may cause dyspnea; however, they will not cause paradoxical chest wall movement unless there is a flail segment present. Tension pneumothorax (D) will classically present with diminished breath sounds on one side, hypotension, and jugular venous distension.

50-year-old man presents to the emergency department unresponsive. He was brought to the hospital by private vehicle and slumped to the ground while walking into the waiting room. Emergency department staff have wheeled him into the resuscitation bay. The patient is unresponsive and pulseless so you initiate cardiopulmonary resuscitation. No other history is obtainable at this time as the family is still in the car. The patient is receiving oxygen via bag-valve mask. When CPR is stopped for a brief moment for a pulse check, you notice the above rhythm on the monitor. What is the most likely diagnosis for this presenting rhythm? Acute coronary syndrome Hypoxia Pulmonary embolism Severe sepsis

Correct Answer ( A ) Explanation: This patient is presenting in cardiac arrest with ventricular tachycardia. Both inside and outside the hospital, sudden cardiac arrest in an adult is attributable to primary cardiac disease in the majority of cases. Among the cardiac causes of arrest, acute coronary syndromes are the most common. Cardiac arrest due to acute coronary syndromes most commonly present in ventricular tachycardia or ventricular fibrillation due to the direct ischemic effects on the myocardium. Management of cardiac arrest begins with assessing the airway, breathing and circulation of the patient and initiating cardiopulmonary resuscitation. In this patient presenting with cardiac arrest due to ventricular tachycardia, along with CPR, defibrillation is indicated. Although different drugs are still indicated according to ACLS guidelines for cardiac arrest, more emphasis should be placed on high quality CPR and defibrillation. There is little evidence that pharmacological interventions impact long term survival in cardiac arrest patients. Among noncardiac causes of arrest, hypoxia (B) is the most common. Other frequent causes include pulmonary embolism (C), tension pneumothorax, pericardial tamponade, severe sepsis (D), and hypovolemia. Pulseless electrical activity (PEA) is the most common presenting rhythm for patients with noncardiac causes of arrest. Asystole is the end rhythm for all causes of cardiac arrest, regardless of etiology. The patient in this question is presenting with ventricular tachycardia and is most likely to have a cardiac rather than a noncardiac cause for arrest.

75-year-old man with a history of hypertension presents with right flank pain. Blood pressure is 180/100 mm Hg and heart rate is 100 beats per minute. A urinalysis dipstick has a small amount of blood. You perform a focused ultrasound assessment and record the video above. What is the sonographic finding? Aortic dissection Hydronephrosis Intrarenal calculus Urinary bladder mass

Correct Answer ( A ) Explanation: This video demonstrates a transverse view of the aorta. The hyperechoic curved line with acoustic shadowing is the vertebral body. Just anterior to the vertebral body is the abdominal aorta in cross-section. Visualized within the lumen of the aorta is a thin hyperechoic line that moves with the cardiac cycle. This line represents the intimal flap of an aortic dissection.Hydronephrosis (B) is often seen in patients with obstructive uropathy. Sonographically, hydronephrosis is identified by visualizing a pronounced hypoechoic (black) area in the renal pelvis as the collecting system dilates. An intrarenal calculus (C) may be seen when evaluating the kidney sonographically as a hyperechoic (white) structure with acoustic shadowing. A urinary bladder mass (D) may be seen on sonographic evaluation as a wall irregularity protruding into the body of the bladder.

64 year-old woman presents to the emergency department complaining of facial pain. She was at a major league baseball game and was struck in the face with a foul ball. She is awake and alert and able to provide you with the history. On physical exam she has extensive swelling and ecchymosis to the nasal bridge and upper lip region. Upon grasping her upper teeth, the nasal complex and maxilla move together as a unit. A CT of the facial bones reveals a fracture extending from the maxilla superiorly to the nasal bridge in a pyramidal shape. What is the most likely diagnosis? Le Fort I fracture Le Fort II fracture Le Fort III fracture Le Fort-Wagstaffe fracture

Correct Answer ( B ) Explanation: A Le Fort II fracture is a complex fracture of the mid-face. It is typically bilateral and pyramidal in shape. The fracture extends from the maxilla superiorly to the nasal bridge, including the lacrimal bones, orbital floor and rim. On physical exam the nasal complex moves with the maxilla when the upper teeth are grasped and rocked. CT of the facial bones will confirm the diagnosis. Management includes pain control and facial trauma consultation for operative repair. A Le Fort I fracture (A) involves a transverse fracture through the maxilla above the roots of the teeth. It may be unilateral or bilateral. Patients often complain of malocclusion. On physical exam the maxilla may be mobile when the upper teeth are grasped and rocked. A Le Fort III fracture (C) is more rare and results in a craniofacial disjunction. These fractures start at the bridge of the nose and extend posteriorly along the medial wall of the orbit, along the orbital floor, through the lateral wall of the orbit and through the zygomatic arch. They often are associated with a CSF leak due to fracture extension through the base of the sphenoid. A Le Fort-Wagstaffe fracture (D) is an avulsion fracture of the anterior cortex of the lateral malleolus.

40-year-old man is brought to the Emergency Department after being assaulted during a bar fight. He has proptosis of the right eye with a measured intraocular pressure of 50 mm Hg. A lateral canthotomy is started. Once the Kelly clamp is released from the lateral canthus, what is the appropriate next step? Clamp the medial canthus Cut the inferior crus of lateral canthal tendon Cut the superior crus of lateral canthal tendon Recheck intraocular pressure

Correct Answer ( B ) Explanation: A lateral canthotomy is performed to relieve intraocular pressure from a retrobulbar hematoma, or postseptal hemorrhage, which creates orbital compartment syndrome and requires immediate decompression. This is an ophthalmologic emergency. The most common cause is blunt trauma to the eye. Abrupt postseptal hemorrhage increases intraocular pressure and decreases optic nerve blood flow, leading to ischemia and vision loss if not corrected in a timely fashion. Exam findings include proptosis, difficulty with extraocular movements, eye pain, decreased visual acuity and elevate intraocular pressure ( > 40 mm Hg). A lateral canthotomy begins with anesthetizing the lateral canthus with lidocaine with epinephrine, followed by crushing it with a straight Kelly clamp for 1-2 minutes. This will also help minimize further bleeding. When the clamp is removed, cut the inferior crus of the lateral canthal tendon. The procedure is successful if intraocular pressure is lowered and pain and visual acuity improve. If not, the superior crus of the lateral canthus can be cut to allow for further decompression. Ophthalmology should be consulted in all cases. There is no indication to clamp the medial canthus (A) when managing a retrobulbar hematoma. One may need to cut the superior crus of the lateral canthal tendon (C) if cutting the inferior crus does not relieve intraocular pressure. Before rechecking intraocular pressure (D), the inferior crus of the rectus sheath should be cut to complete the lateral canthotomy.

Violation of which of the following structures defines a penetrating neck injury? Mylohyoid muscle Platysma Sternocleidomastoid muscle Subcutaneous tissue

Correct Answer ( B ) Explanation: A penetrating neck injury is defined by penetration of the platysma, a thin muscle stretching from the facial muscles to the thorax that demarcates superficial from deep injuries of the neck. Injuries superficial to the platysma are not life-threatening. Neck injuries deep to the platysma are emergent due to the numerous aerodigestive, vascular, and neurological structures that run through the neck. Most vital structures are contained within the anterior triangle of the neck, which is defined by the borders of the sternocleidomastoid muscle, inferior mandible, and the midline of the neck. The anterior triangle is divided into three zones. Classically, zone II injuries undergo surgical exploration while zone I and zone III injuries undergo further diagnostic testing with bronchoscopy, esophagoscopy, or CT angiogram. In reality, the trajectory of penetrating neck injuries is often difficult to determine and the decision to explore the wound surgically or obtain more diagnostic testing should be driven by the patient's hemodynamic status, the appearance of the wound, and surgical consultation. The mylohyoid (A) is a paired muscle that forms the floor of the oral cavity and connects the mandible to the hyoid bone. The sternocleidomastoid (C) originates at the manubrium and clavicle and inserts at the temporal bone of the skull. It forms borders of both the anterior and posterior cervical triangles and is deep to the platysma. Penetration of these muscles, as well as subcutaneous tissue (D), does not define a penetrating neck injury.

21-year-old man presents to the emergency department with bleeding from his nose after he was in an altercation at the bar tonight. Initial examination is remarkable for swelling and slight deformity to the bridge of the nose without any lacerations. The man is bleeding from his right naris. You attempt to stop the bleeding with compression, use of intranasal vasoconstrictors and cautery without success. You decide to place anterior packing which controls the bleeding. No other facial injuries are noted on examination or imaging. What is the most appropriate management of this patient? Begin prophylactic antibiotics and discharge the patient to follow up with otolaryngology in three days Consult interventional radiology for embolization of a bleeding anterior nasal artery Consult otolaryngology to evaluate the patient in the emergency department as the patient will need emergent surgery No further emergency department intervention; discharge the patient to follow up with otolaryngology in three days

Correct Answer ( D ) Explanation: Treatment of nasal injuries begins with the same management as any soft-tissue injury and includes compression to control bleeding. Irrigation is necessary for contaminated wounds. Many nasal fractures can be diagnosed clinically and the management is conservative with pain medicine, rest, ice and head elevation. If nasal bleeding continues, identification of the source of bleeding is advisable. Treatment of continued epistaxis may include silver nitrate cautery which is successful in the majority of cases. If bleeding continues anterior nasal packing may be required. It is recommended that packing be removed in approximately three to four days. Use of prophylactic antibiotics in the setting of nasal packing is controversial and the recommendation at this time is to discharge the patient without antibiotics to see otolaryngology in two to three days. It has been previously recommended that all patients with nasal packing receive prophylactic antibiotics upon discharge from the emergency department due to concern for toxic shock syndrome. However, the incidence of toxic shock syndrome after packing for epistaxis is unknown and in one study of patients who underwent nasal surgery and had packing placed the incidence was as low at 16 cases per 100,000 patients. No data exists to indicate that the use of prophylactic antibiotics will reduce the incidence of toxic shock syndrome. In fact, there are likely more complications from the use of antibiotics in this setting. Therefore, beginning prophylactic antibiotics and discharging the patient to follow up with otolaryngology in three days (A) is incorrect. No evidence currently exists to support early nasal fracture reduction and management. Treatment is conservative and includes rest, ice, pain control and head elevation. Even if nasal packing is required to control hemorrhage consulting otolaryngology to evaluate the patient in the emergency department (C) is not correct. Follow up evaluation with an otolaryngologist is recommended after swelling has subsided. If bleeding in the nose continues after compression, cautery and anterior nasal packing attempts, then one must consider a posterior bleed. Posterior packing may need to be placed and the patient should be admitted because patients with posterior packing in place are at higher risk for hypoxia, hypoventilation and dysrhythmias. If posterior packing strategies fail, a consult to interventional radiology may be necessary for embolization. However, consulting interventional radiology for embolization of a bleeding anterior nasal artery (B) is not appropriate in this case.

80-year-old man presents to the emergency department with right knee pain and swelling for three days. Physical examination shows mild swelling in the right knee without overlying erythema. The knee joint feels warm to the touch. It is tender and painful during passive range of motion. You determine an arthrocentesis needs to be performed. Results show synovial fluid yellow in color with 5000 WBC/mm3, and polymorphonuclear cell count of 60%. Gram stain and crystal evaluation are negative. Of the following, what is the most likely diagnosis in this patient? Gout Lyme arthritis Osteoarthritis Septic arthritis

Correct Answer ( B ) Explanation: Arthrocentesis and synovial fluid analysis are helpful in determining the cause of arthritis and should be performed in all patients with signs and symptoms suggestive of inflammation in the joint without a known underlying cause. The most important reason to perform synovial fluid analysis is to evaluate for septic arthritis. Synovial fluid analysis is also helpful for definitively diagnosing inflammatory arthropathies such as gout, pseudogout, reactive arthritis and Lyme arthritis. Results of synovial fluid analysis can be divided into non-inflammatory, inflammatory, septic and hemorrhagic. The color of the fluid and the nucleated white blood cell count helps lead the clinician to one diagnosis versus another. Inflammatory effusions such as gout, pseudogout, reactive arthritis and Lyme arthritis show a nucleated white blood cell count of > 3000 cells/mm3. Based on synovial fluid analysis, the patient in this question appears to have an inflammatory arthritis. Of the options given, gout and Lyme arthritis are inflammatory. Lyme arthritis is a manifestation of late-phase Lyme disease and appears to be mediated by an autoimmune process rather than by direct infection. Lyme arthritis can be differentiated from gout and pseudogout by the lack of crystal formation in the synovial fluid. Confirmation of Lyme arthritis depends on immunoglobulin M and G serology as inflammatory arthritis without crystal formation can also be reactive arthritis. Gout (A) is also an inflammatory arthritis. Diagnosis is confirmed by the presence of monosodium urate crystals which are not present in this patient's synovial fluid analysis. Similar to gout, pseudogout is diagnosed by calcium pyrophosphate dihydrate crystals in synovial fluid analysis. Generally non-inflammatory effusions, such as those in osteoarthritis (C), show a nucleated white blood cell count of < 3000 cells/mm3. The likelihood of septic arthritis (D) increases as the number of nucleated white blood cells increases. Generally, septic effusions show a nucleated white blood cell count of at least > 25,000 cells/mm3 and some consider a cut-off of > 50,000 cells/mm3. However, immunocompromised patients, elderly patients and those with mycobacteria or Neisseria arthritis may have somewhat lower nucleated white blood cell counts.

Which of the following signs or symptoms is seen most frequently in the initial evaluation of a patient with a basilar skull fracture? Cerebrospinal fluid otorrhea Hemotympanum Mastoid ecchymosis Periorbital ecchymosis

Correct Answer ( B ) Explanation: Basilar skull fractures are linear fractures that occur anywhere along the skull base from the cribriform plate to the occipital condyles. The most common basilar skull fracture involves the petrous portion of the temporal bone, the auditory canal, and the tympanic membrane. There can be an associated dural tear leading to cerebrospinal fluid (CSF) otorrhea or rhinorrhea. The presence of a leak provides a means for introduction of infection putting these patients at increased risk for development of meningitis. Given the close proximity of these fractures to the middle meningeal artery, patients are also at increased risk for extra-axial hematomas, particularly epidural hematomas. Hemotympanum is the most frequent finding on physical examination. The tympanic membrane will appear blue to purple in color. Other symptoms include evidence of a CSF leak, mastoid ecchymosis (Battle's sign), periorbital ecchymosis (raccoon eyes), vertigo, decreased hearing, and seventh nerve palsy. Diagnosis is made via noncontrast CT of the head. Neurosurgical consultation and admission is warranted. The decision to give prophylactic antibiotics in patients with evidence of a CSF leak is somewhat controversial and should be made in conjunction with the neurosurgeon. Mastoid ecchymosis, also known as Battle sign (C) and periorbital ecchymosis, known as raccoon eyes (D) are both signs concerning for a basilar skull fracture, but are not commonly seen during the acute evaluation as they take 1-3 days to appear. CSF otorrhea (A) is another concerning sign for a basilar skull fracture, but is often difficult to diagnose. The leaks may develop one to many days after the initial injury. Fluid can be collected and analyzed for the presence of ß transferrin which is only found in cerebrospinal fluid. The majority of CSF leaks resolve within one week without complication.

53-year-old man presents to the ED after being struck in his right eye while playing basketball. He has decreased vision and flashing lights in the affected eye. On exam, his extraocular muscles are functioning appropriately. Globe is normal in shape and fluorescein staining is negative. You obtain an ocular ultrasound as seen above. Which of the following is the most likely diagnosis? Globe perforation Retinal detachment Retrobulbar hematoma Traumatic hyphema

Correct Answer ( B ) Explanation: Blunt injury to the retina may result in hemorrhage, a tear, or detachment. Symptoms include floaters from bleeding, flashing lights from stimulation of retinal neurons, and visual field cuts or decreased visual acuity. However, visual acuity may be normal if the macula is not involved. Retinal detachments are important to recognize early as they may progress to full detachments and loss of vision if untreated. The ultrasound image shows a retinal detachment with associated vitreous hemorrhage. As the patient looks side to side, the vitreous hemorrhage has a swirling effect affectionately called the washing machine sign.The detached retina, on the other hand, undulates in a serpiginous fashion because it remains tethered to the optic disk and surrounding healthy retina. Globe perforation (A) is also associated with trauma. Sonographically, the globe may be decreased in size, indicating loss of vitreous. Buckling of the sclera may be seen posteriorly and vitreous hemorrhage is usually present. Retrobulbar hematoma (C) is seen as an echolucency (dark) just posterior to the globe. Sonographically, there is distortion to the posterior aspect of the eye. Traumatic hyphema (D) should be seen on physical exam as blood in the anterior chamber and is not a sonographic diagnosis.

A 50-year-old man presents to the Emergency Department with severe swelling and pain of his left forearm. He sustained a puncture wound to this area three days ago while working on a constructions site. He has a diminished left radial pulse and severe pain with passive extension of his fingers. Which muscle in the deep volar compartment is at risk for ischemia? Flexor carpi ulnaris Flexor digitorum profundus Palmaris longus Supinator

Correct Answer ( B ) Explanation: Compartment syndrome is a musculoskeletal emergency characterized by increased pressure within a confined space outlined by fascial planes that leads to tissue death and neurovascular compromise if not managed in a timely manner. Trauma is a common cause, including penetrating injuries, fractures, or crush injuries. However, external forces such as a tight dressing or cast may also precipitate compartment syndrome. Envenomation and high-pressure injection injuries are other causes. Intracompartmental hemorrhage or edema may lead to compression of neurovascular structures, leading to limb ischemia and necrosis. This occurs when tissue pressure or pressure from an external source exceeds perfusion pressure. Compartment syndrome is a clinical diagnosis, characterized by pain out of proportion to exam findings, pulselessness, pallor, poikilothermia (coolness to touch), paresthesias and exacerbation of pain with passive extension of the digits. Compartment pressures may be measured directly to aid in the diagnosis. Normal compartment pressure is 0-10 mm Hg. Compartment syndrome occurs when intracompartmental pressure exceeds 30 mm Hg, or when the difference in the diastolic blood pressure and intracompartmental pressure (called the "delta pressure") is less than 30 mg Hg. Most cases of compartment syndrome occur in the lower leg, followed by the forearm. The forearm is divided into four fascial compartments. The superficial volar compartment contains the flexor carpi ulnaris, flexor carpi radialis, flexor digitorum superficialis, palmaris longus and pronator teres muscle. The deep volar compartment contains the flexor digitorum profundus, flexor pollicis longus and pronator quadratus muscles. The lateral compartment, or mobile wad of Henry, contains the brachioradialis, extensor carpi radialis brevis and extensor carpi radialis longus muscles. The dorsal compartment contains the extensor muscles including the extensor carpi ulnaris, extensor pollicis longus, extensor digitorum, and supinator muscles. The treatment of choice for compartment syndrome is early decompression. This can occur by way of emergent fasciotomy or cast cutting or removal. The flexor carpi ulnaris (A) and the palmaris longus (C) muscles are located in the superficial volar compartment of the forearm. The supinator muscle (D) is located in the dorsal (extensor) compartment.

27-year-old man is involved in a motor vehicle collision in which he was the unrestrained driver. He presents to the Emergency Department with significant facial trauma. A CT of his face shows a Le Fort type III fracture. Which of the following structures is directly affected? Central maxilla Ethmoid bone Hard palate Mandibular condyle

Correct Answer ( B ) Explanation: Complex facial fractures may be classified into the Le Fort system of fracture patterns. Three types exist. Type I Le Fort fractures are transverse fractures through the base of the maxilla and often involve the roots of the teeth. It may be unilateral and bilateral. The upper teeth may be grasped and rocked independent of the midface, indicating discontinuity with the maxilla. Type II Le Fort fractures are often bilateral and are characterized by pyramidal fractures that extend through the body of maxilla, orbital floor, nasal bones, hard palate, and lacrimal bones. The nasal unit and maxilla may be grasped and move as a unit relative to the orbital complexes and lower face. Type III Le Fort fractures are considered complete craniofacial disjunction as the fracture extends from the nasal bridge traveling posteriorly along the ethmoid bone, which comprises the medial orbital wall, and continues through the inferior and lateral orbital walls and frontozygomatic suture. The sphenoid bone is often involved with type III fractures, and cerebrospinal fluid (CSF) leak may occur. CSF may also leak from involvement of the cribriform plate of the ethmoid bones. Le Fort fractures are best seen with computed tomography. Depending on the extent of the injury, Le Fort fractures often require management of a multidisciplinary team and may necessitate consultation to otolaryngology, plastic and reconstructive surgery, dentistry and neurosurgery if a CSF leak is present. A type II Le Fort fracture involves the central maxilla (A), nasal bridge, lacrimal bones, hard palate (C), and orbital floor. It often occurs bilaterally and in a pyramidal pattern. Fractures of the mandibular condyle (D) are not included in the Le Fort classification of midface fractures.

22-year-old man presents to the emergency department with foot pain after landing on his foot improperly while playing basketball last night. Using the image, what is the diagnosis in this patient? AChopart's fracture BJones fracture CLisfranc fracture DPseudo-Jones fracture

Correct Answer ( B ) Explanation: Fractures of the proximal fifth metatarsal pose an important diagnostic challenge. A difference in millimeters of location can lead to a vastly different prognosis and treatment plan. Suboptimal treatment can lead to delayed union, re-injury and chronic disability. The fifth metatarsal can be divided into three parts: the tuberosity, the metaphysis and the proximal diaphysis. There are three basic types of fractures including tuberosity avulsion fractures, acute proximal diaphyseal fractures, and stress fractures of the proximal diaphysis. The easiest way to distinguish among the fractures is to locate the medial tip of the fracture line and compare its location to the intermetatarsal joint. Styloid avulsion fractures exit proximal to the intermetatarsal joint. These are called pseudo-Jones fractures. Acute proximal diaphyseal fractures extend into or towards the intermetatarsal joint. These are called Jones fractures. Stress fractures typically exit or extend distal to the intermetatarsal joint. The patient in this question has an acute proximal diaphyseal fracture, or Jones fracture. In comparison to an avulsion fracture, or pseudo-Jones injury, the Jones fracture is more likely to fail conservative management. Therefore, immobilization in a posterior splint and quick referral to an orthopedist within three to five days is indicated. Chopart's fracture (A) and dislocation is a dislocation of the midtarsal joints of the foot including the talonavicular and calcaneocuboid joints and associated fractures of the calcaneus, cuboid and navicular bones. The radiograph in this question does not show fractures of the calcaneus, cuboid and navicular bones. A Lisfranc fracture (C) is an injury of the foot in which one or more of the metatarsal bones are displaced from the tarsus. The radiograph in this question does not show a Lisfranc injury. As already mentioned, a pseudo-Jones fracture (D) is a styloid avulsion fracture of the fifth metatarsal. These fractures generally heal very well and do not require surgical intervention. The fracture line in this patient is more distal and is consistent with a Jones fracture rather than a pseudo-Jones fracture.

Which of the following measurements can be used as a marker to assess for the presence of increased intracranial pressure? Intraocular pressure Optic nerve sheath diameter Posterior chamber depth Retinal thickness

Correct Answer ( B ) Explanation: Ocular ultrasound can be used in the emergency setting to aid in the evaluation of globe trauma, vitreous hemorrhage, intraocular foreign bodies, and retinal detachment. It has also been found to be an easy method of assessing intracranial pressure. A normal optic nerve sheath diameter is up to 5 mm in adults, 4.5 mm in children, and 4.0 mm in infants. Using a high resolution linear array transducer, the diameter of the optic nerve sheath is measured 3 mm behind the globe. The average of 3 measurements is taken for each eye. An optic nerve sheath diameter > 5 mm in adults predicts an intracranial pressure of > 20 mm Hg with a sensitivity and specificity of 88-94% and 76-93% respectively. Elevated intraocular pressure (A) can be seen in acute glaucoma but does not correlate with intracranial pressure. Posterior chamber depth (C) can be decreased in the setting of a ruptured globe. An increase in retinal thickness (D), indicating macular edema, can be due to numerous causes such as diabetes, uveitis and retinal vein occlusion, but does not correlate with increased intracranial pressure.

48-year-old man presents to the emergency department with vision changes and right eye pain after accidentally getting hit with a bat while playing baseball tonight. He denies any other injuries. Examination is remarkable for periorbital swelling, ecchymosis and tenderness. Vision is 20/40 in each eye. You note that the patient cannot elevate his right globe when asked to look up but the remainder of the eye exam is normal. Computed tomography performed shows an orbital blowout fracture of the right orbit with entrapment. What is the appropriate management of this patient? Apply ice for swelling, recommend non-steroidal anti-inflammatory agents for pain and discharge to follow up with ophthalmology in two days Apply ice for swelling, recommend non-steroidal anti-inflammatory agents for pain, start prophylactic antibiotics and discharge to follow up with ophthalmology in two days Consult ophthalmology for evaluation in the emergency department and admit to medicine for intravenous antibiotics Consult ophthalmology for evaluation in the emergency department as this patient will need emergent surgery

Correct Answer ( B ) Explanation: Orbital blowout fractures are defined as fractures involving the floor of the orbit into the maxillary sinus or ethmoid bone. They are frequently caused by a direct blow to the orbit and eye. Patients with simple orbital blowout fractures without eye injury can be discharged home with appropriate ophthalmology referral. This is true even in the case of entrapment. Initial management includes applying ice intermittently to the periorbital area for 48 hours, elevation of the head of the bed while sleeping, nasal decongestants, avoidance of blowing the nose so as to not increase intraorbital pressure, non-steroidal anti-inflammatory agents to help with swelling, and broad-spectrum antibiotics such as amoxicillin-clavulanate to prevent infection. The use of prophylactic antibiotics can be controversial in this setting as there is limited evidence to support or refute their use in an orbital blowout fracture. The thought is that a fracture that involves the sinus poses a risk for infectious complications. Even though evidence is lacking, the recommendation at this time is to prescribe prophylactic antibiotics. Therefore, applying ice for swelling, recommending non-steroidal anti-inflammatory agents for pain and discharging to follow up with ophthalmology in two days (A) without antibiotics would not be correct. Consulting ophthalmology for evaluation in the emergency department and admitting to medicine for intravenous antibiotics (C) would be incorrect as simple blowout fractures do not require emergent intervention unless serious eye injury or decreased vision is found. Along the same lines, consulting ophthalmology for evaluation in the emergency department as this patient will need emergent surgery (D) is incorrect. Indications for performing early surgery include patients with evidence of orbital compartment syndrome, such as acute changes in visual acuity, severe increase in pain, elevated intraocular pressure, afferent pupillary defect and proptosis. Other patients who may be candidates for early surgery include those who complain of seeing flashing lights, enophthalmos or substantial herniation into the maxillary sinus.

What is the most common complication after permanent pacemaker placement? Infection Pacemaker syndrome Pneumothorax Superior vena cava syndrome

Correct Answer ( B ) Explanation: Pacemaker syndrome, occurring in up to 20% of patients after permanent pacemaker placement, is the result of loss of atrioventricular synchrony and the presence of retrograde ventriculoatrial conduction. It is most commonly seen in patients with VVI pacemakers. Patients may present with syncope or near-syncope, heart failure, fatigue, exercise intolerance, dizziness as well as other nonspecific symptoms. While the majority of patients with pacemaker syndrome have mild symptoms that they are able to adapt to, one-third of patients have severe symptoms requiring intervention. In patients with a VVI pacemaker, upgrading to a dual chamber pacemaker or lowering the pacing rate of the VVI pacemaker improves or relieves symptoms. Infection (A) occurs in less than 2% of patients after permanent pacemaker placement and is often due to S. aureus or S. epidermidis. Pneumothorax (C) is also very uncommon after pacemaker placement, affecting less than 1% of patients. While some degree of venous obstruction is common with pacemaker placement, symptoms of superior vena cava syndrome (D), which include edema, pain and venous engorgement of the arm ipsilateral to the site of pacemaker placement, is rare.

Which of the following is considered a contraindication to the administration of succinylcholine? Renal failure with a serum potassium of 3.8 mEq/L Spinal cord injury sustained one month earlier Stroke with onset 24 hours prior to arrival Third degree burns sustained one hour prior to arrival

Correct Answer ( B ) Explanation: Succinylcholine is a depolarizing agent that exerts its effects by binding non-competitively with acetylcholine (ACh) receptors on the motor endplate of myocytes and causing sustained depolarization. It is rapidly active typically producing intubating conditions within 60 seconds of administration. The duration of action is 6 to 10 minutes with full recovery within 15 minutes. Succinylcholine binds to ACh receptors throughout the body including the heart, where it serves as a negative chronotrope, especially in children. Pretreatment with atropine is considered in this population. Fasciculations are common in over 90% of patients. Hyperkalemia can occur within minutes after administration and may be severe or fatal; however, the patient's vulnerability to succinylcholine-induced hyperkalemia does not become significant until at least five days after the inciting injury or burn. Succinylcholine is the agent of choice for rapid sequence intubation in acute burn, trauma, stroke, spinal cord injury and sepsis if given less than five days after onset. Acute denervation syndromes, such as stroke and spinal cord injury, are stabilized after 3-6 months. Therefore, succinylcholine can be administered after this period. Spinal cord injury sustained one month earlier is too early to give succinylcholine due to the risk of severe hyperkalemia. Chronic and indefinite denervation syndromes, such as multiple sclerosis or amyotrophic lateral sclerosis, have inherent risk at the onset of the illness that continues indefinitely. Thus, succinylcholine should be avoided in these patients. Renal failure with a serum potassium of 3.8 mEq/L (A) is not a contraindication for administration of succinylcholine. If hyperkalemia is present or there are ECG changes suggesting hyperkalemia, then succinylcholine should be avoided. Stroke onset 24 hours prior to arrival (C) is not a contraindication to administering succinylcholine because it is within five days of onset of the condition. Third degree burns sustained one hour prior to arrival (D) are not a contraindication to administering succinylcholine because it is within five days of onset of the condition.

19-year-old man presents to the ED by ambulance after striking a car with his motorcycle. His vital signs are BP 85/55 mm Hg, HR 140, RR 20, and pulse oximetry 98% on room air. Two liters of normal saline are administered and a repeat blood pressure is 80/60 mm Hg. A Focused Assessment with Sonographyfor Trauma (FAST) exam is performed as seen above. Which of the following is the most appropriate next step in management? CT scan Exploratory laparotomy Immediate thoracotomy Pericardiocentesis

Correct Answer ( B ) Explanation: The FAST scan is positive for a significant amount of free fluid in the left upper quadrant (splenorenal recess). In the setting of trauma, free intraperitoneal fluid is presumed to be blood. Despite the administration of 2 L normal saline, the patient remains hypotensive. Therefore, in this clinical scenario (blunt abdominal trauma, unstable vital signs, and positive intraperitoneal fluid), the patient requires emergent laparotomy. A CT scan (A) is appropriate only in the setting of blunt abdominal trauma and a positive FAST exam when vital signs are stable. Emergent thoracotomy (C) is indicated only for blunt abdominal trauma when the patient loses vital signs on arrival in the ED or during resuscitation. Though hypotensive, this patient still has a detectable blood pressure. Pericardiocentesis (D) is a life-saving (albeit temporizing) measure used when cardiac tamponade is present. There is no evidence of tamponade in this case.

The Emergency Medical Treatment and Active Labor Law of 1986, or EMTALA, requires hospitals to provide which of the following? Care to all patients Interpreters for all patients in a timely manner Screening exam, competent ED physicians and appropriate stabilization Screening exam, stabilization process, appropriate transfer process

Correct Answer ( B ) Explanation: The Lisfranc joint (tarsometatarsal joint complex) is comprised of the articulations of the bases of the first three metatarsals with the cuneiforms and the fourth and fifth metatarsals with the cuboid, joined together with multiple ligaments. The stability of the joint is essential for function, making the timely and accurate diagnosis of injury, whether sprain, dislocation, or fracture-dislocation, crucial in order to avoid development of osteoarthritis and long-term disability. Lisfranc injuries are relatively uncommon and can occur via excessive rotational forces, axial loads, or crush injuries. While MVCs are responsible for most Lisfranc injuries, they can also occur in the setting of an athletic injury or seemingly trivial falls. Patients present with severe pain in the midfoot and inability to bear weight. Ecchymosis on the plantar surface of the foot is often considered pathognomonic for Lisfranc injuries. Anterior-posterior, lateral, and oblique radiographs should be ordered initially. Findings may be subtle. The medial border of the second metatarsal should align with the medial aspect of the middle cuneiform and the medial border of the fourth metatarsal should align with the medial border of the cuboid. Any widening or malalignment > 1 mm in these areas is considered pathological. In addition, any fractures of the proximal metatarsals should be concerning for a possible Lisfranc injury. Initial treatment consists of immobilization with a short-leg cast, elevation, and pain control. The patient should be non-weight bearing. Orthopedic consultation is necessary. A Jones fracture (A) is a fracture of the meta-diaphyseal junction of the proximal fifth metatarsal of the foot. Navicular fractures (C) are the most common midfoot fractures. Because of the pattern of blood supply to the navicular bone, it is at increased risk for avascular necrosis when injured. An isolated nondisplaced metatarsal fracture (D) can be treated with immobilization with a below-the-knee walking cast. This patient has both clinical and radiographic evidence that is most consistent with a Lisfranc injury.

4-year-old girl falls while playing on the playground and injures her right ankle. She is brought to the Emergency Department and an X-ray is obtained, as shown above. What is the most likely diagnosis? Salter-Harris type II fracture Salter-Harris type III fracture Salter-Harris type IV fracture Salter-Harris type V fracture

Correct Answer ( B ) Explanation: The Salter-Harris fracture classification describes pediatric long bone fractures that involve the physeal (growth) plate. Five types exist and are characterized based on the involvement of the physis. Type I fractures involve a shearing mechanism through the physis with separation or disruption of the metaphysis and epiphysis from the physis. Type I fractures are often radiographically silent, so a high index of clinical suspicion is required. Type II fractures include fractures of the metaphysis. Type III fractures include fractures of the epiphysis and are considered intra-articular. Orthopedic consultation is required as reduction and fixation may be needed to achieve proper alignment and minimize growth disturbances. Type IV fractures involve both the metaphysis and epiphysis with the fracture extending through the physis. Type V fractures are caused by axial loading or compressive force to the physis, leading to a crush-type injury of the growth plate. Salter-Harris type II fractures (A) involve the metaphysis (shaft), not the epiphysis, of the long bone. Salter-Harris type IV fractures (C) involve fractures of both the metaphysis and the epiphysis in a "through-and-through" fracture pattern through the growth plate. Salter-Harris type V fractures (D) are caused by compression of the physis but do not involve a fracture fragment of the metaphysis or epiphysis.

23-year-old man is brought into the Emergency Department by police after being shot in the chest. He loses pulses upon arrival and a resuscitative thoracotomy is performed. Which of the following structures along the heart surface should be identified before performing a pericardiotomy? Long thoracic nerve Phrenic nerve Thoracic duct Vagus nerve

Correct Answer ( B ) Explanation: The decision to perform a resuscitative thoracotomy in traumatic cardiac arrest should be made on a case-by-case basis, and is usually determined by the mechanism of injury and the presence or absence of signs of life (e.g., pupillary response, spontaneous ventilation, carotid pulse, extremity movement, cardiac activity). Survival rates in cases of penetrating thoracic trauma are greater than that of blunt thoracic or abdominal trauma. Injuries due to stab wounds have higher success rates than those from gunshot wounds. A left-anterolateral approach is preferred to optimize direct visualization of the heart and access the great vessels. After the skin is prepped with iodine, an incision is made along the fifth rib from the sternum to the posterior axillary line. The intercostal muscles are then either sharply or bluntly dissected, followed by placement of rib spreaders to expose the intrathoracic space. The heart is identified and the pericardium should be inspected for evidence of effusion. The phrenic nerve is a tendon-like structure that runs along the surface of the heart and innervates the diaphragm muscle bilaterally. It may be easily injured during a resuscitative thoracotomy if it is not identified. Injury to the phrenic nerve results in diaphragm paralysis and severe respiratory compromise. In unilateral injuries, this causes hemidiaphragm elevation and intrathoracic compression by intra-abdominal contents when lying supine. Scissors are used to open the pericardium parallel to the phrenic nerve from the apex to the base of the heart. Any cardiac injuries should be repaired as they are identified. The aorta is then identified and cross clamped to prioritize blood flow to the coronary and cerebral vessels. If vital signs are regained after the thoracotomy, patients may be transported to the operating room. The long thoracic nerve (A) is located along the lateral chest wall and innervates the serratus anterior muscle. It is at risk for injury during procedures such as chest tube placement, thoracotomy, thoracic surgery or radical mastectomy. The thoracic duct (C) is a lymphatic structure that lies in the posterior chest wall and drains into the junction of the left internal jugular vein and left subclavian vein. The vagus nerve (D) supplies visceral innervation to the thoracoabdominal contents. It is located in the neurovascular bundle in the neck and thoracic inlet and begins to branch near the large vessels of the aortic arch.

In a study, 10% of patients developed a cardiovascular disease after receiving medication X, compared to 20% of patients who developed the disease in the control group. What is the number needed to treat (NNT) to prevent one cardiovascular event by taking medication X? 1 10 20 5

Correct Answer ( B ) Explanation: The number needed to treat (NNT) measures the effectiveness of a health intervention. The NNT is the number of patients that need to be treated for one to benefit compared with a control. It is defined as the inverse of the absolute risk reduction (ARR). The higher the NNT, the less effective is the treatment. In the above scenario, the risk of developing the disease after the treatment is 0.1 (10%), whereas, the risk is 0.2 (20%) if in the control group. The ARR is 0.2 minus 0.1, which is 0.1. Therefore, the NNT is 10, the inverse of ARR.

22-year-old man presents with a change in his vision after being hit in the left eye. On examination, his visual acuity is 20/20 OD and 20/150 OS. Intraocular pressure is 14 mm Hg OD and 15 mm Hg OS. Extraocular movements are intact bilaterally. His ocular ultrasound is shown above. What is the likely diagnosis? Intraocular foreign body Lens dislocation Retinal detachment Ruptured globe

Correct Answer ( B ) Explanation: The ocular ultrasound confirms a diagnosis of lens dislocation. The lens has an echogenic biconvex shape with an anechoic center and typically lies between the anterior and posterior chamber. With trauma or in patients with a predisposition for lens dislocation (e.g., Marfan syndrome, tertiary syphilis), the lens can dislocate anteriorly or posteriorly. Patients will present with altered vision and monocular diplopia. Trembling or shimmering of the iris after rapid eye movements (iridodonesis) can be a sign of lens dislocation. Intraocular pressure may be elevated if the dislocated lens prevents outflow of aqueous humor. Ocular ultrasound will show a biconvex reflective mass in the posterior or anterior chamber which moves independently with eye movements. If due to trauma, there also may be evidence of vitreous hemorrhage. An intraocular foreign body (A) will have a bright echogenic profile and shadowing or reverberation may be present. An echogenic undulating membrane in the most posterior aspect of the eye that moves with eye movements is characteristic of a retinal detachment (C). Signs of a ruptured globe (D) on ultrasound include scleral buckling, disruption of the normal size and shape of the globe and anterior chamber collapse. Care must be taken when performing ocular ultrasound in the case of a suspected globe rupture to avoid placing pressure on the eye. This can be done by using a large amount of ultrasound gel so that the transducer does not actually touch the eye.

18-year-old woman presents to the emergency room with severe clavicle pain. She is a soccer player and collided with another player during her game. She has bruising and swelling over her clavicle without skin tenting. Above is an X-ray of her clavicle. Which of the following risk factors does this patient have for nonunion of her injury Age Gender Location of fracture Percent of displacement

Correct Answer ( B ) Explanation: This patient has a middle one-third clavicle fracture, the most common type. This is followed by lateral one-third fractures and end with the most rare, medial one-third fractures. Lateral one-third fractures are commonly due to direct blows to the shoulder, while medial one-third fractures are due to direct trauma to the mid-chest. Middle one-third fractures can occur in seemingly innocuous falls or from large motor vehicle or sporting collisions. Risk factors associated with clavicle fracture nonunion include the elderly, high mechanism of injury, female gender, medial one-third fractures, severely comminuted fractures (over 2 cm of shortening), and over 100% displacement. Treatment for most includes immobilization with a sling, early passive range of motion, and routine orthopedics follow-up. Skin tenting, neurovascular compromise, and open fractures require immediate surgical consultation. Follow-up with orthopedics within the next one to three days is recommended for patients with a higher risk of nonunion. This patient's age (A) is not a risk factor for nonunion, however, elderly patients are at greater risk. The location of the fracture (C) may increase the risk of nonunion if the fracture is in the medial one-third of the clavicle. This patient's fracture is angulated but not displaced. Patient's with over 100% displacement (D) are at greater risk of nonunion.

Which of the following is the most common location of traumatic aortic injury? Aortic arch between the left common carotid and the left subclavian arteries Aortic isthmus immediately distal to the left subclavian artery Ascending aorta immediately proximal to the brachiocephalic artery Descending aorta immediately distal to the diaphragm

Correct Answer ( B ) Explanation: Traumatic aortic injury can occur with blunt trauma to the chest, typically with a high-energy mechanism and rapid deceleration. Motor vehicle collisions are the most common cause, though falls from a height can result in aortic injury as well. Most traumatic aortic injuries occur at the isthmus of the aorta, immediately distal to the branching of the left subclavian artery. This is thought to be due to the isthmus being the transition zone between the mobile ascending aorta and the more fixed thoracic aorta. Most aortic injuries are fatal before hospital arrival. For surviving patients, CT or transesophageal echocardiography are used for diagnosis, with bedside transesophageal echocardiography being preferable for hemodynamically unstable patients. Management begins with blood pressure and heart rate control. The majority of aortic injuries require surgical repair, but injuries restricted to the intima may be managed medically. Though aortic injuries proximal to the brachiocephalic artery (C), at the aortic arch (A), and in the descending aorta (D) are possible, they are less common than injuries at the isthmus of the aorta.

32-year-old woman presents to the Emergency Department with cardiopulmonary resuscitation in progress after being struck by a car while crossing the street. She has already been intubated by EMS prior to arrival. She is 29 weeks pregnant and her uterine fundus is 4 cm above the umbilicus. No maternal pulse is palpable. Fetal heart tones are present with a heart rate of 130 beats/minute. Which of the following is the most important next definitive step to take? Discontinue chest compressions Perimortem cesarean section via low transverse incision Perimortem cesarean section via midline vertical incision Uterine displacement

Correct Answer ( C ) Explanation: A perimortem cesarean section is indicated in cases of maternal demise with the presence of fetal heart tones and the uterine fundal height is above the umbilicus, thus improving the chances of fetal viability. There has been debate whether C-section is indicated if there are no fetal heart tones present; however, some studies have indicated improved maternal outcomes and rare neonatal survival after C-section. The most experienced provider should make a midline vertical incision from the epigastrium to the pubic symphysis down to the uterus. Another vertical incision is made through the uterine musculature to deliver the fetus. If the placenta is obscuring access to the fetus, this may be incised as well, being careful not to incur injury to the fetus. Ideally, the procedure should be performed within five minutes of maternal arrest to provide a favorable neurologic outcome of the fetus. Survival is highest with fetuses ≥ 26 weeks gestation. Do not discontinue cardiopulmonary resuscitation (A) if fetal viability is highly likely in the presence of maternal demise. Measures should be taken to perform adequate resuscitation to the mother to improve fetal survival. Uterine displacement (D) should be performed if the uterus is at or above the umbilicus in order to maximize the cardiac return; however, this should not delay the time-sensitive emergent delivery of a viable fetus in the perimortem period. A low transverse incision (B) is more appropriate for elective C-section and does not provide adequate exposure or access to the fetus for perimortem delivery.

45-year-old man with a history of hypercholesterolemia presents with acute onset of anterior chest pain, nausea, and diaphoresis. His ECG is distinct for upsloping ST elevation in AVR (≥ 1 mm) with ST depressions and tall T waves in the precordial leads. What is the most likely diagnosis? Occlusion of the left circumflex artery Occlusion of the left main coronary artery Occlusion of the proximal left anterior descending artery Occlusion of the right coronary artery

Correct Answer ( C ) Explanation: Acute anterior wall myocardial infarction is most commonly characterized on ECG as ST segment elevation along the precordial leads representing occlusion of the left anterior descending artery. The de Winter ECG pattern is a less frequently seen ECG finding (found in approximately 2% of acute LAD occlusions) that represents an acute anterior infarction without precordial ST elevation. Key features include tall, prominent symmetric T waves in the precordial leads with upsloping ST segment depression greater than 1 mm at the J point. ST segment elevation may be seen in aVR. The de Winter pattern may evolve into the classic anterior STEMI morphology or it may persist until reperfusion is achieved. Many consider this pattern to be a "STEMI equivalent" and, as such, patients who present with symptoms concerning for acute coronary syndrome and a de Winter pattern on ECG should receive emergent reperfusion therapy. Occlusion of the left circumflex artery (A) results in a lateral myocardial infarction which is seen on ECG as ST elevation in the I, aVL, and V5-6 with reciprocal ST depression in III and aVF. ST segment elevation in aVR > V1 with ST segment depression in I, II and V4-6 is predictive of occlusion of the left main coronary artery (B). The majority of inferior infarctions are caused by occlusion of the right coronary artery (D). Classic ECG findings with inferior MI include ST elevation in II, III and aVF with reciprocal ST depression in aVL.

83-year-old woman presents to the Emergency Department from her nursing home after a witnessed fall from standing. She complains of left hip pain. She is non-ambulatory at baseline and uses a wheelchair for mobility. Range of motion of the left hip does not elicit significant pain and she is not requiring parenteral analgesia. A pelvis X-ray is obtained and shown above. Which of the following is the most appropriate management for this patient? Admission for operative fixation Closed reduction under moderate sedation Discharge with weightbearing as tolerated Emergent orthopedic consultation

Correct Answer ( C ) Explanation: Because the pelvis is a rigid ring-like structure, it is most likely to fracture in more than one place. However, isolate single-bone fractures within the ring are possible. Isolated pubic ramus fractures are the most common pelvic fractures and are considered stable fractures. They are especially common in elderly people who fall. Unilateral superior and inferior pubic rami fractures may also occur from this mechanism and are generally considered stable as well. However, if significant displacement is present, further investigation for an additional fracture in the pelvic ring (e.g., acetabulum, sacrum, sacroiliac joint) should occur. Other fractures that do not typically disrupt the pelvic ring and usually do not need orthopedic repair include isolated fractures of the coccyx, sacrum, anterior superior iliac spine, anterior inferior iliac spine, ischial tuberosity, body of the ischium and iliac wing. These injuries are typically managed conservatively with pain control, crutches, appropriate weight-bearing instructions and outpatient orthopedic follow-up. Therefore, it is appropriate for this patient to be discharged with weightbearing as tolerated. Isolated pubic ramus fractures are considered stable and typically do not involve vascular compromise or concomitant pelvic organ injury, so admission for operative fixation (A) is not indicated. Closed reduction under moderate sedation (B) is required in a timely fashion for hip dislocations in order to prevent neurovascular compromise and avascular necrosis. This patient does not have a hip dislocation. Emergency orthopedic consultation (D) is unnecessary for isolated pubic ramus fractures, and patients may follow up with orthopedics on an outpatient basis.

70-year-old man presents to the Emergency Department with twelve hours of urinary retention and suprapubic pressure. Bedside ultrasound of the bladder and kidneys show no hydronephrosis and an estimated bladder volume of 750 mL. Urinalysis and laboratory values are unremarkable. Which of the following is the most appropriate next step in management? Apply steady suprapubic pressure Obtain aortic angiography Place urethral catheter Refer for outpatient hemodialysis

Correct Answer ( C ) Explanation: Common causes of acute urinary retention in men include benign prostatic hyperplasia, prostate cancer, phimosis, paraphimosis, meatal stenosis, urethral strangulation, and prostatitis. The most common presentation is an elderly man with urinary retention for several hours as a result of benign prostatic hyperplasia (BPH). Narrowing the differential diagnosis is aided by clinical presentation, vital signs, urinalysis, serum laboratory tests (e.g., electrolytes, renal function panel) and bedside ultrasound to assess for bladder stones, hydronephrosis, or intra-abdominal masses. In an elderly male with acute urinary retention secondary, the next step is to place a urethral catheter for bladder decompression. This may be difficult because of urethral stricture, prostate enlargement, or postsurgical bladder neck contractures. Urology should be consulted before catheter placement if the patient had a recent urologic procedure. Suprapubic catheterization may be indicated if multiple attempts at urethral placement fail or if there are contraindications for urethral placement. Patients with uncomplicated BPH (e.g., no urinary tract infection, normal renal function) may be discharged home with the catheter and a leg bag with close urologic follow up. Applying steady suprapubic pressure (A) is not recommended and may in fact be harmful in patients with acute bladder outlet obstruction due to BPH. Obtain aortic angiography (B) or CT with contrast in elderly patients with abdominal, flank or back pain, as symptomatic abdominal aortic aneurysms (AAA) can masquerade as renal colic or gastroenteritis. While a ruptured AAA can cause hydronephrosis, it is unusual for it to cause abrupt urinary retention. It is unnecessary to refer for outpatient hemodialysis (D) in patients with uncomplicated acute urinary retention from BPH and normal renal function.

Which of the following patients would benefit most from the use of noninvasive positive pressure ventilation? A 9-year-old girl with an acute asthma exacerbation and an oxygen saturation of 88% on room air An anxious elderly woman with chronic obstructive pulmonary disease with an oxygen saturation of 86% on room air who is excessively coughing and unable to sit still in bed An obese elderly man who presents with bilateral crackles, diffuse pedal edema and an oxygen saturation of 82% on room air An unresponsive patient with an oxygen saturation of 75% on room air

Correct Answer ( C ) Explanation: Early utilization of noninvasive ventilatory management, particularly in patients with chronic obstructive pulmonary disease and congestive heart failure, has been shown to decrease the need for intubation, intensive care unit admissions, length of stay and hospital admissions. Additionally, use of noninvasive ventilation in these patients improves mortality. In addition to its usefulness as a bridge to intubation in a patient with respiratory distress, noninvasive ventilation has also shown benefits for patients with pneumonia, asthma, interstitial lung disease and, most recently, blunt chest trauma. Appropriate patients for noninvasive ventilatory management include those in acute respiratory distress who are cooperative and alert, are an appropriate fit for the mask or device used for ventilatory support and are without copious secretions or vomiting. A 9-year-old girl with an acute asthma exacerbation and an oxygen saturation of 88% on room air (A) is incorrect because this patient would likely benefit from nasal cannula oxygen and bronchodilator therapy. Additionally, evidence supporting the use of noninvasive ventilation in pediatrics is currently lacking. An anxious elderly woman with chronic obstructive pulmonary disease with an oxygen saturation of 86% on room air who is excessively coughing and unable to sit still in bed (B) is incorrect because the patient does not appear to be cooperative and will not likely be able to synchronize her respiratory effort with that provided by the pressure support machine. An unresponsive patient with an oxygen saturation of 75% on room air (D) is incorrect because the use of noninvasive ventilation requires a patient to be alert and cooperative to avoid aspiration.

34-year-old, right hand dominant man presents with pain and swelling of his left index finger. The patient works as a painter and was using an industrial paint sprayer and sprayed his left index finger. At first, his finger was not painful but he has developed increasing pain and swelling. The finger is mildly swollen but no laceration, erythema, or warmth is noted. Range of motion is full and sensation and capillary refill are normal. In addition to splinting and elevation, which of the following represents the best management course? Admission to medical service for intravenous antibiotics Discharge with urgent orthopedics follow up within 72 hours Emergent hand surgery consultation Performance of bedside incision and drainage

Correct Answer ( C ) Explanation: High pressure injection injuries are associated with high morbidity. Devices which spray liquids at high pressure can inject foreign material deep into soft tissue. This causes an inflammatory response and fibrosis which varies based upon the material injected. Paint and paint thinner cause an earlier, more intense inflammatory response and result in a high number of amputations. Grease injuries, on the other hand, cause a less intense inflammatory response and fewer amputations, but more often cause granuloma and fistula formation with loss of function. Symptoms may be minimal immediately after the injury but swelling, vascular compromise and tissue necrosis develop in a matter of hours. X-rays may reveal the extent of spread of radio-opaque material and can show subcutaneous edema. Patients with these injuries should be splinted, given tetanus prophylaxis and broad spectrum antibiotics. The affected extremity should be elevated. Emergent hand surgery consultation is necessary as most cases require extensive surgical decompression and debridement. This patient should not be admitted to a medical service for intravenous antibiotics (A) without hand surgery consultation as he will likely require surgical debridement. Discharge with urgent orthopedics follow up within 72 hours (B) is inappropriate, as the inflammatory response is likely to progress in a matter of hours. A delay in treatment would likely lead to significant morbidity. Performance of bedside incision and drainage (D) is inappropriate as this patient will require surgical exposure of the deep tissues of the hand to ensure adequate washout.

You arrive on the scene of a mass-casualty incident after a bomb has exploded in a crowd of people. You initiate the simple triage and rapid treatment (START) technique. You approach an adolescent male lying on the ground moaning in pain, with a macerated and obvious amputation at his mid-right thigh. There is dusky skin at the edge of the amputation and active hemorrhage from the stump. Which triage color category should this patient receive? Black Green Red Yellow

Correct Answer ( C ) Explanation: In mass-casualty events, rescue personnel use the START technique to perform a quick assessment of respiration, perfusion, and mental status. The only patient care interventions during this process are opening of an obstructed airway and direct pressure on an obvious external hemorrhage. A color-coded triage tag is used to classify the injured. A red tag is placed if the patient is breathing over 30 breaths per minutes (respirations), capillary refill over 2 seconds (perfusion), unable to follow simple commands (mental status), or has significant active hemorrhage. An amputation of an extremity, therefore, will receive a red tag due to the active hemorrhage. Black tags (A) are placed on victims who have no respirations after head tilt. Green tags (B) are placed on victims able to walk (walking wounded). Green-tagged individuals are asked to move away from the immediate casualty area. Yellow tags (D) are placed on patients who cannot immediately ambulate from the scene but who have a respiratory rate less than 30, a palpable radial pulse, and can follow simple commands.

21-year-old chef recently cut his hand with a knife. The presence of which of the following clinical findings suggests a diagnosis of infectious flexor tenosynovitis? Finger fixed in slight extension Pain with passive flexion of the digit Percussion tenderness along the flexor tendon Positive Finkelstein's test

Correct Answer ( C ) Explanation: Infectious flexor tenosynovitis is characterized clinically by four cardinal signs referred to as the Kanavel signs. These include (1) percussive tenderness along the flexor tendon, (2) circumferential swelling of the digit, (3) a finger fixed in slight flexion to minimize pain, and (4) pain with passive extension of the digit. It is most commonly caused by penetrating trauma to the digit, including bites from mammals or humans. Staphylococcus aureus is the most common organism causing infection. Early diagnosis is crucial, and antibiotic therapy should be initiated immediately. Surgical consultation is prudent as deep fascial irrigation and debridement or exploration is often needed. A finger fixed in slight extension (A) suggests a diagnosis other than flexor tenosynovitis, in which the finger is fixed in slight flexion. Pain with passive flexion (B) is not a characteristic of flexor tenosynovitis, but rather pain with passive extension. A positive Finkelstein's test (D) is diagnostic of De Quervain tenosynovitis, which affects the lateral forearm tendons that attach to to the thumb.

31-year-old man was the restrained passenger involved in a head-on motor vehicle collision. He presents to the emergency department complaining of right knee pain. On physical examination, there is significant deformity of the right knee with an intact dorsalis pedis pulse. An X-ray is obtained and shown above. Which of the following is considered a hard sign of vascular injury? Diminished sensation Nearby open fracture Palpable thrill Weak distal pulses

Correct Answer ( C ) Explanation: Knee dislocation is an orthopedic emergency, and timely reduction is essential to restore neurovascular function. Dislocation may occur in a variety of planes, including anterior (most common), posterior, lateral, medial, or rotatory. Severe ligamentous damage often occurs due to high mechanical forces causing the injury. Therefore, the knee joint is very unstable and may have relocated spontaneously prior to presentation. Providers should have a high index of suspicion for a knee dislocation if the knee is unstable in multiple planes. Hard signs of vascular injury should be identified. These include limb ischemia, the absence of a pulse, pulsatile bleeding, rapidly expanding hematoma, an audible bruit, or a palpable thrill. An ankle-brachial index of < 0.9 is also clinically concerning for a vascular injury. A popliteal artery injury is the most devastating complication of knee dislocation, so early diagnosis is critical. Orthopedic surgery and vascular surgery consultation are often needed for further evaluation and management of this injury. Diminished sensation (A) or other neurologic abnormality in the affected extremity should raise suspicion for neurovascular compromise but is considered a soft sign of vascular injury. A nearby open fracture (B) may cause a vascular injury but does not itself indicate the presence of a vascular injury. Any bony injury or penetrating wound proximate to a vascular structure is considered a soft sign of vascular injury. Weak distal pulses (D) compared to the contralateral extremity is considered a soft sign of vascular injury. Factors other than injury may cause diminished pulses, such as thromboembolic disease, atherosclerosis or soft tissue edema.

22-year-old woman presents to the Emergency Department for fever, sore throat and odynophagia. Her vital signs are T 102.3ºF, HR 109 beats/minute, RR 12 breaths/minute, BP 119/75 mm Hg. Examination reveals a left pharyngeal mass displacing the uvula to the right. There is no neck swelling, erythema or stiffness. Which of the following is the most appropriate next step in the management of this patient? Emergent tonsillectomy Intravenous clindamycin Needle aspiration Referral to otolaryngology

Correct Answer ( C ) Explanation: Peritonsillar abscesses (PTAs) are purulent collections anterior to the tonsil proper that lie within the peritonsillar space confined by the tonsillar capsule, the superior constrictor muscle and the palatopharyngeus muscle. Most PTAs are polymicrobial in nature, but Fusobacterium necrophorum is a commonly isolated organism. Patients report sore throat, fever, malaise, odynophagia, and possibly otalgia. Exam findings include a muffled voice, uvula deviation away from the abscess, tender cervical adenopathy, trismus, and dehydration. Diagnosis is clinical, but computed tomography may be indicated if there is suspicion of spread beyond the peritonsillar space. Approximately 90% of patients with a peritonsillar abscess are treated effectively with a single needle aspiration. Other less-used treatments include incision and drainage, and for select patients, immediate tonsillectomy. Patients should also be treated with a course of oral antibiotics that cover oral flora. Examples include penicillin VK plus metronidazole, or clindamycin if penicillin allergy is present. Emergent tonsillectomy (A) is only indicated in select patients with PTA, such as those with sleep apnea, recurrent tonsillitis or recurrent abscesses. A 10-day course of oral antimicrobials against group A Streptococcus species and anaerobes should be initiated for patients with a PTA. Intravenous clindamycin (B) is not indicated in uncomplicated cases. Referral to otolaryngology (D) is unnecessary for a simple peritonsillar abscess that is amenable to needle aspiration.

38-year-old man presents to the Emergency Department with a posterior hip dislocation after striking his left knee on the dashboard during a motor vehicle crash. He has weakness with dorsiflexion of the left foot. Which nerve has likely been damaged? Femoral Obturator Sciatic Tibial

Correct Answer ( C ) Explanation: Posterior hip dislocations are almost always the result of motor vehicle crashes. A seated vehicle occupant typically has the hip adducted, flexed, and internally rotated at the time of impact. As the knee strikes the dashboard, the force is transmitted through the femoral shaft to the femoral head. With enough force, the femoral head dislocated posteriorly. A patient with a posterior hip dislocation typically holds the hip flexed, adducted, and internally rotated. The neurovascular exam should focus on the sciatic nerve and femoral vessels. Sciatic palsy is present in approximately 10% of patients with hip dislocation and most commonly involves the peroneal branch. The most sensitive clinical sign is weakness of the extensor hallucis longus. Other signs include weakness of dorsiflexion and numbness or tingling over the dorsum of the foot. The femoral nerve (A) innervates the muscles of the anterior thigh and allows hip flexion and knee extension. The obturator nerve (B) innervates the medial thigh and allows adduction of the thigh. The sciatic nerve, once it reaches the popliteal fossa, bifurcates into the common peroneal nerve, which innervates the anterior lower leg and allows dorsiflexion of the foot, and the tibial nerve (D), which innervates the posterior lower leg and allows plantarflexion of the foot.

27-year-old woman presents to the Emergency Department with full thickness burns to her chest, back, and extremities measuring approximately 25% total body surface area. She weighs 80 kg. Two large-bore intravenous lines are established. How much intravenous fluid should she receive per hour for the first eight hours of resuscitation? 300 mL/hour 480 mL/hour 500 mL/hour 600 mL/hour

Correct Answer ( C ) Explanation: The Parkland formula is used to calculate the amount of fluid required to adequately resuscitate a burn victim in the first 24 hours. For patients with partial thickness or full thickness burns, the formula is as follows: 4 x (weight in kilograms) x (percentage total body surface area affected). Half of this volume is given over the first eight hours; the remainder is given over the next sixteen hours. For this patient, the equation works as follows: 4 x 80 kg x 25 = 8000. She would require 8000 mL of fluid over 24 hours. However, this volume is divided in half, so that the first half (i.e., 8000 ÷ 2 = 4000 mL) is given over eight hours. Therefore, 4000 ÷ 8 = 500. Therefore, this patient needs 500 mL/hour over the first eight hours of resuscitation. Volumes of 300 mL/hour (A) and 480 mL/hour (B) would be inadequate to resuscitate this patient in the first eight hours. The other option, 600 mL/hour (D), would place the patient at risk for volume overload.

28-year-old woman presents to the ED after being involved in a high speed MVC. Her BP is 90/40, pulse 110, respiratory rate 22 and oxygen saturation is 95%. A bedside FAST ultrasound is performed as seen above. All other scans of the FAST are normal. Which of the following statements is true? The patient has a large hemothroax and chest tube thoracostomy should be placed emergently The patient has a pericardial effusion and should be prepped for emergent pericardiocentesis The patient has evidence of intraperitoneal fluid and operative management of suspected liver injury should be considered This FAST is considered negative

Correct Answer ( C ) Explanation: The above FAST images show evidence of intraperitoneal free fluid in Morrisons pouch and perihepatically.Patients with this finding and who have unstable vital signs are candidates for operative management. A FAST (Focused Assessment with Sonography for Trauma) examination is a rapid, bedside ultrasound scan commonly used in the ED to assess for hemoperitoneum and hemopericardium in the setting of trauma. Traditionally this includes views of the splenorenal and hepatorenal spaces, pelvis and heart. The eFAST (extended FAST) also includes views of both sides of thorax and looks for underlying pneumothorax or hemothorax. The advantages of FAST include speed, ease of use, accuracy, portability, repeatability and non-invasive nature. Additionally, there is no radiation risk or contrast needed. The main limitations include operator error, inability to detect retroperitoneal bleeding and inability to determine exact cause of positive findings. The ability to interpret acquired FAST images is an essential skill for EM physicians to possess. The findings shown above are considered positive; although they are negative for pericardial effusion, there is intraperitoneal fluid noted (D). FAST can detect pericardial effusion (B), although this is not what is shown in these images. Similarly, eFAST can detect hemothorax (A) however this is not seen in the ultrasound above.

23-year-old man presents to the emergency department with acute onset of dizziness. His blood pressure is 80/40 mm Hg and his ECG is shown above. Which of the following is the next best step in management? Esmolol Intravenous fluids Synchronized electrical cardioversion Vagal maneuv

Correct Answer ( C ) Explanation: The treatment of unstable supraventricular tachycardia is synchronized cardioversion. Supraventricular tachycardia is commonly classified into atrioventricular nodal reentrant tachycardia and atrioventricular reentrant tachycardia. In atrioventricular nodal reentrant tachycardia or AVnRT, the reentrant circuit originates within the AV node. In atrioventricular reentrant tachycardia or AVRT, a reentrant circuit is formed between an accessory pathway and the AV node. The electrical impulse moves down the AV node through the ventricles and then up through the accessory pathway back to the AV node completing the circuit. The goal of treatment is to restore normal electrical conduction by breaking the reentrant circuit. Synchronized cardioversion is advised in patients with altered mental status, hypotension, or signs of shock. Esmolol (A) may be considered in stable patients who have failed treatment with adenosine. Intravenous fluids (B) would be indicated in patients with hypotension secondary to hypovolemia. In a patient with supraventricular tachycardia, the treatment should focus on improving heart rate which will improve end-diastolic ventricular volume, resulting in improved cardiac output. Vagal maneuvers (D) may be attempted in stable patients with supraventricular tachycardia. Overall, these maneuvers are minimally effective and are not the best treatment option in an unstable patient.

A 35-year-old woman presents to the ED with fever and abdominal pain. Vital signs are BP 100/65 mm Hg, HR 110 and T 38.3C. A focused ultrasound assessment is performed as seen in the video. What is the most likely diagnosis? Ascending cholangitis Biliary colic Cholecystitis Choledocolithiasis

Correct Answer ( C ) Explanation: The ultrasound demonstrates multiple findings of acute cholecystitis.These include thickening of the gallbladder wall and the presence of gallstones and pericholecystic fluid. There was likely a sonographic Murphys sign during the ultrasound study. The sonographic Murphys sign differs from that found on physical examination (arrest of inspiration on deep palpation of the right upper quadrant) and is positive when the point of maximal tenderness is identified in the right upper quadrant, when the gallbladder is identified on the ultrasound screen. In this way, one can be certain that the gallbladder is the tender structure as it is being directly visualized. Ascending cholangitis (A) is not a sonographic diagnosis but one made clinically in patients with abdominal pain, fever, and jaundice. Choledocolithiasis (D) is defined by a stone in the common bile duct. This is not usually seen sonographically. However, choledocholithiasis will cause the common bile duct to dilate (> 6 mm), which can be measured sonographically. Biliary colic (B) describes the pain caused by gallstones, particularly those in the neck of the gallbladder. When biliary colic occurs in the setting of fever, leukocytosis, pericholecystic fluid, and a thickened gall bladder wall, the diagnosis of cholecystitis is made.

38-year-old man presents to the ED after he was assaulted. The patient reports he was punched and kicked across his thorax. The area of most pain is in his left lateral chest. On arrival in the ED, his blood pressure is 130/95 mm Hg, heart rate is 100 beats per minute, temperature is 36.8C, respiratory rate is 20 breaths per minute, oxygen saturation is 100% on room air. Physical exam reveals tenderness to palpation along the left lateral costal margin and left upper quadrant of the abdomen. There are no peritoneal signs and no crepitus noted along the chest wall. A supine portable chest X-ray shows no acute intrathoracic process. A bedside FAST exam is performed as seen below. Repeat blood pressure is 140/90 with a heart rate of 95. Which of the following is the most appropriate next step in the management of this patient?

Correct Answer ( C ) Explanation: The ultrasound shows an anechoic collection in the subphrenic space consistent with peritoneal free fluid.In the setting of trauma, free fluid collections are presumed to be hemoperitoneum. Free fluid in the left upper quadrant most often collects in the subphrenic space. A patient with blunt abdominal trauma who has stable vital signs, and hemoperitoneum on FAST exam, requires a CT of the abdomen and pelvis with IV contrast to determine the source of bleeding as well as extent of injury. The most likely source of hemoperitoneum is solid-organ injury (spleen in this case). Solid-organ injuries are frequently managed nonoperatively (A) with hemodynamic monitoring and serial abdominal exams but, prior to such decision making, a CT is needed. Following CT imaging, this patient may be placed in observation (B). However, regardless of the injury pattern, six hours is unlikely to be a sufficient observation period. All unstable patients with hemoperitoneum require immediate laparotomy (D). This patients vital signs have remained stable, so he should undergo a CT scan of the abdomen to further evaluate his injuries.

85-year-old woman with a past medical history of cerebral vascular accident presents to the ED from a long term nursing facility with the chief complaint of fever. The patient is aphasic and unable to provide further history. Physical exam is unremarkable other than baseline hemiplegia. Vitals on arrival are significant for heart rate 137, blood pressure 70/40, and temperature 39.1℃. You order labs, blood cultures, lactate level, a chest X-ray, urinalysis, as well as begin broad spectrum antibiotics and a 30 mL/kg normal saline bolus. After fluids, the blood pressure is now 80/45. What is the next best step in the management of this patient? Administer an additional 1 liter normal saline bolus Begin norepinephrine infusion through a peripheral IV Place a central venous line and begin a norepinephrine infusion Place a central venous line and begin a phenylephrine infusion

Correct Answer ( C ) Explanation: This patient meets systemic inflammatory response syndrome (SIRS) criteria with a heart rate greater than 90 and a temperature greater than 38℃. While no apparent bacterial source is noted in the question vignette, people with a history of stroke are at higher risk of infections, particularly pneumonia due to aspiration and urinary tract infections due to limited self hygiene. Sepsis (SIRS + suspected or proven infection) is the working diagnosis in this vignette. According to the Surviving Sepsis Campaign, patients with suspected sepsis who are hypotensive or have a lactate concentration greater than 4 mmol/L should receive a 30 mL/kg bolus of crystalloid solution. This patient is in septic shock as she is persistently hypotensive despite adequate fluid resuscitation. The next best step in the management of this patient is initiation of vasopressors, with a targeted mean arterial pressure of greater than 65. Norepinephrine is currently accepted as the first line agent for septic shock as it increases mean arterial pressure through peripheral vasoconstriction without increasing heart rate or cardiac output. Dopamine can also be used, but it has a higher risk for inciting arrhythmias and is also associated with higher rates of mortality. Vasopressin and Epinephrine are second line agents which can be used if the first agent is insufficient to raise the MAP to greater than 65. Pressors are most effective when given through a central venous line. Central lines can be placed in the internal jugular, subclavian, or femoral veins. These are typically triple lumen catheters which are 16 or 22 centimeters long and allow multiple medications to be pushed through the same line. Emergency physicians usually place femoral or internal jugular central lines using ultrasound guidance, which has been shown to increase the rate of successful line placement while decreasing the rate of adverse events. Additional fluids (A) are unlikely to improve the blood pressure as the patient was adequately resuscitated with 30 mL/kg bolus. Beginning norepinephrine through a peripheral IV (B) is generally discouraged due to the risk of local tissue ischemia around the IV catheter if the medication extravasates. Phenylephrine (D) is not considered a first or second line agent for septic shock.

45-year-old woman with a history of dialysis-dependent renal failure presents to the emergency department complaining of shortness of breath. Her vital signs are 95/60 mm Hg, HR 105, RR 24, and pulse oximetry is 97% on room air. A bedside cardiac ultrasound is performed, as seen above. What is the earliest sonographic finding of this patients diagnosis? AInterventricular septal flattening BLeft atrial diastolic collapse CRight atrial diastolic collapse DRight ventricular diastolic collapse

Correct Answer ( C ) Explanation: This patients bedside cardiac ultrasound shows a large pericardial effusion with evidence of right atrial diastolic collapse, the earliest sonographic finding of cardiac tamponade. Thisoccurs during the period of end-diastole to early systole when the RA pressure is minimal.

35-year-old woman presents to the ED with fever, chills, cough, and chest pain for 3 days. Vital signs are BP100/65 mm Hg, HR 100 beats per minute, temperature 38.3C. A focused ultrasound assessment is performed as seen above. Which of the following is the most likely diagnosis? Endocarditis Hepatitis Pericarditis Pneumonia

Correct Answer ( C ) Explanation: This ultrasound demonstrates a subxiphoid view of the heart. At the top of the screen is the liver with the heart in the center of the screen. The bright white line surrounding the heart is the pericardium. The pericardium is fibrous so it appears bright white (hyperechoic). In this video, there is a black (hypoechoic) stripe of fluid between the grey myocardium and the bright white pericardium posteriorly, evidence for a pericardial effusion. The combination of fever, chest pain, and a pericardial effusion on ultrasound is highly suggestive of pericarditis in an otherwise healthy individual. Endocarditis (A) will be seen on ultrasound as vegetations or thickening of the valves. The corresponding video clip shows no abnormal valves. However, these findings are often subtle and can be overlooked by a novice sonographer. Transesophageal echocardiography is often needed to make this diagnosis, especially when left-sided valves are involved. In the acute phase, hepatitis (B) may demonstrate no sonographic findings, and the liver may appear normal. In chronic hepatitis, the normal hepatic parenchyma is replaced with scar tissue, resulting in a shrunken and hyperechoic (white) appearing liver. Pneumonia (D) may present sonographically as consolidated lung tissue with a pleural effusion. The pleural effusion may be simple or complex if empyema is present. This video demonstrates fluid in the pericardial space, not the pleural space.

32-year-old man with a history of intravenous drug use presents with localized discomfort on the right upper arm. An ultrasound of the area is shown above. What is the most likely diagnosis? Abscess Cellulitis Retained foreign body Superficial venous thrombosis

Correct Answer ( C ) Explanation: Ultrasound can be used in the evaluation and management of soft tissue disorders such as cellulitis, abscess and retained foreign bodies. Ultrasound has been shown to have a sensitivity of over 90% in detecting soft tissue foreign bodies such as wood, metal, and plastic greater than 4-5 mm in length. Lower frequency probes (e.g., 3.5 MHz) are able to detect foreign bodies at greater depths, whereas a higher frequency probe (e.g., 12.5 MHz) is better for more superficial foreign bodies. Foreign bodies will appear as a hyperechoic foci with acoustic shadowing extending distally. The presence of a hyperechoic rim, or "halo sign" indicates a surrounding abscess. An abscess (A) is seen as an anechoic or hypoechoic spherical collection. Occasionally, septae or gas can be seen within the area. An ultrasound of cellulitis (B) will show hyperechoic fat lobules in edematous fluid and has a cobblestone appearance. Superficial venous thrombosis (D) appears on ultrasound as a noncompressible vein. The vein may also appear thickened or inflamed.

20-year-old man is brought to the Emergency Department in respiratory distress after sustaining a penetrating injury inferior to his right scapula. He has distended neck veins and absent right sided breath sounds. His trachea is deviated to the left. His vital signs are HR 122 beats/minute, BP 87/56 mm Hg, RR 32 breaths/minute, and oxygen saturation 92% on a nonrebreather face mask. What is the most likely diagnosis? Atelectasis Cardiogenic shock Pulmonary contusion Tension pneumothorax

Correct Answer ( D ) Explanation: A tension pneumothorax is a life-threatening condition that should be diagnosed and managed promptly. It should be a clinical diagnosis (i.e., prior to obtaining a chest X-ray). A tension pneumothorax occurs when air that enters the thoracic cavity between the lung and the chest wall accumulates and creates mass effect by compressing mediastinal structures and great vessels toward the contralateral side. Increased intrapleural pressure is caused by a valve-like effect in which air enters the pleural space but is unable to escape. When the vena cava and right atrium are compressed, diastolic function is impaired, followed by worsening cardiac output and eventually shock. Physical exam findings include tracheal deviation to the contralateral side, diminished breath sounds on the side of the pneumothorax, distended neck veins and hypotension. Management involves immediate needle decompression with a large-bore (e.g., 12- or 14-gauge angiocath) into the ipsilateral second intercostal space in the mid clavicular line or the fourth or fifth intercostal space in the mid axillary line. Atelectasis (A) is collapse of lung parenchyma due to poor aeration and may be due to obstruction (e.g., malignancy, mucus plug) or poor patient positioning. It is unlikely to be the cause of hypotension and respiratory distress, especially in the setting of a penetrating injury. Cardiogenic shock (B) is secondary to a primary insult to the heart itself, such as occurs in acute myocardial infarction, dysrhythmias or papillary muscle rupture. A pulmonary contusion (C) is caused from blunt or penetrating thoracic trauma and involves a parenchymal hemorrhage and edema. Associate rib fractures are common. Onset is usually insidious rather than abrupt following trauma. Breath sounds may include rales or rhonchi but are not absent. Tracheal deviation is not a feature of pulmonary contusion.

23-year-old woman is brought to the hospital by EMS for altered mental status. According to EMS, the patient was found by her family at home confused and agitated on the floor. Vital signs enroute were HR 145, BP 184/100, R 24, T 103.7°F, oxygen saturation 98% on room air and finger stick for glucose 100 mg/dL. The patient has a history of depression and migraine headaches and has been drinking alcohol heavily lately due to a recent break-up with her boyfriend. On arrival to the emergency department, vital signs are unchanged. You find a young woman who is agitated in bed and trying to grab at the monitoring devices attached to her. She is diaphoretic. She makes sounds but does not make any sense. Pupils are 6 mm in size and equal. She moves her arms equally in a spasmodic fashion every few minutes. Her legs are stiff and when you hyperflex at the ankle you see rhythmic beating. What is the most likely diagnosis in this patient? Alcohol withdrawal syndrome Anticholinergic poisoning Neuroleptic malignant syndrome Serotonin syndrome

Correct Answer ( D ) Explanation: A toxidrome is a group of signs and symptoms that constitute a basis for a poisoning syndrome. Toxidromes are best evaluated by examining a patient's mental status, pupils, lungs, vital signs, bowel sounds, bladder, skin, and neurological status. The important findings in this patient include agitation and delirium, tachycardia, hypertension, hyperthermia, diaphoresis, mydriasis, and clonus. Many different toxidromes present with these signs and symptoms but the rhythmically beating feet, or clonus, should lead one to suspect serotonin syndrome. Serotonin syndrome is a life-threatening adverse drug reaction to a combination of drugs that increase central serotonin neurotransmission. Antidepressants such as monoamine oxidase inhibitors, selective serotonin reuptake inhibitors and cyclic antidepressants are commonly associated with serotonin syndrome. Patients generally present with agitation, altered mental status, restlessness, diaphoresis, hyperthermia, tachycardia, hypertension, mydriasis, tachypnea, incoordination, muscle rigidity, hyperreflexia and myoclonus. Myoclonus is a common finding and is an important distinguishing feature because it is rarely seen in other conditions that mimic serotonin syndrome. The vast majority of cases occur in patients taking therapeutic doses of the drugs, but serotonin syndrome usually occurs 2-24 hours after either increasing the dose of one medication or adding a second serotonergic medication to the treatment regimen. The patient in this question has signs and symptoms characteristic of alcohol withdrawal syndrome (A) including hypertension, tachycardia, hyperthermia, agitation, delirium, mydriasis, and hyperreflexia, however, clonus is not found in alcohol withdrawal syndrome. Symptoms characteristic of anticholinergic poisoning (B) include hyperthermia, tachycardia, hypertension, agitation, delirium, and mydriasis, but diaphoresis in this patient goes against anticholinergic poisoning. Generally anticholinergic poisoned patients would have dry skin. Additionally, clonus is not present in anticholinergic poisoning. Finally, serotonin syndrome can easily be confused with neuroleptic malignant syndrome (C), however, clonus is a defining feature of serotonin syndrome and is the test used to differentiate serotonin syndrome from neuroleptic malignant syndrome.

n 32-year-old man is brought in by EMS after being the unrestrained driver in a motor vehicle collision. His vitals on arrival are HR 150, BP 140/90, RR 25. He is noted to have the above finding on physical examination. Which of the following structure is most likely to be injured? Cribriform plate Occipital bone Orbital plate Temporal bone

Correct Answer ( D ) Explanation: Basilar skull fractures are those which involve any of the five bones comprising the skull base, including the cribriform plate, the occipital bones, the sphenoid bones, the squamous and petrous portions of the temporal bones, and the orbital plate of the frontal bones. The most common basilar skull fracture involves the temporal bone, the external auditory canal, and the tympanic membrane. Hemotympanum, as noted with this patient, is likely to be seen in this setting. Due to common involvement of the temporal bone, basilar skull fractures are often associated with extra-axial hematomas, especially epidural hematomas due to damage of the middle meningeal artery as it crosses the temporal bone. Cerebrospinal fluid leaks due to dural tears are also possible, resulting in CSF otorrhea or rhinorrhea. Although the cribriform plate (A), the occipital bone (B), and the orbital plate (C) of the frontal bone are all structures in the basilar skull, they are less likely to be fractured than the temporal bone.

27-year-old man presents after a brief syncopal episode while sitting at a meeting at work. EMS was called and he has been awake and alert since leaving his office. He denies any current symptoms. Vital signs are unremarkable. His electrocardiogram is shown above. What pertinent finding would be most concerning in his family history? Atrial fibrillation Early coronary artery disease Hypercoagulable disorder Sudden cardiac death at age 40 years

Correct Answer ( D ) Explanation: Brugada syndrome, first described in 1992, is an ECG abnormality that is associated with a high incidence of sudden cardiac death in patients with structurally normal hearts. It results from an inherited disorder of the sodium channels and is more common in males. The mean age of sudden death is 41 years. Diagnosis is dependent not only on presence of the specific ECG abnormality but the clinical scenario as well. Brugada sign is described as coved ST segment elevation greater than 2 mm in leads V1-V3 followed by a negative T wave. ECG changes may transient. Patients with this ECG finding and one of the following clinical criteria - documented ventricular fibrillation or polymorphic ventricular tachycardia, family history of sudden cardiac death under the age of 45 years, similar ECG finding in family members, inducible ventricular tachycardia with electrical stimulation, syncope or nocturnal agonal respiration - are at high risk for sudden cardiac death due to dysrhythmia. Definitive management is placement of an automatic implanted cardiac defibrillator. Patients who are asymptomatic with Brugada sign on ECG but no clinical criteria of Brugada syndrome should be referred to a cardiologist for further evaluation. Patients with Brugada syndrome are also at increased risk for atrial dysrhythmias, including atrial fibrillation (A). The presence of atrial fibrillation has been associated with increased disease severity and a higher incidence of ventricular fibrillation. A family history of atrial fibrillation alone would not increase the risk of Brugada syndrome however. Early coronary artery disease (B) is not linked to Brugada syndrome. A family history of a hypercoagulable disorder (C) or known venous thromboembolism (VTE) in a family member increases a patient's risk of developing VTE as well. ECG findings associated with a pulmonary embolism include sinus tachycardia, right ventricular strain and new incomplete right bundle branch block. This patient's ECG is consistent with Brugada sign.

In patients who are not in cardiac arrest, which of the following most accurately confirms tracheal placement of an endotracheal tube? Chest wall movement with delivery of tidal volume Fogging of the endotracheal tube Presence of equal breath sounds bilaterally Qualitative colorimetric end tidal carbon dioxide detector

Correct Answer ( D ) Explanation: Confirmation of proper placement of an endotracheal tube is essential as unrecognized misplaced intubation results in significant morbidity and mortality. Carbon dioxide monitors have become commonplace in the emergency department and allow for measurement, either qualitatively or quantitatively, of the partial pressure of carbon dioxide in the exhaled breath. The most commonly used qualitative device is the colorimetric end tidal carbon dioxide detector. Consisting of a treated piece of litmus paper that changes color from purple to yellow when exposed to carbon dioxide, this device has become standard in the use of confirming placement of an endotracheal tube. When used in patients who are not in cardiac arrest, it has a sensitivity and specificity of nearly 100% for tracheal placement. It should be used with caution in patients in cardiac arrest or in low perfusion states as the sensitivity is variable due to the lower concentration of exhaled CO2 Chest wall movement (A) can be produced by both esophageal and tracheal tubes and is not recommended as confirming appropriate placement. Fogging of the endotracheal (B) occurs in over 80% of esophageal intubations, and likewise, should not be used for confirmation. The presence of equal breaths sounds (C) can incorrectly identify proper placement in approximately 15% of cases.

36-year-old man presents to the emergency department complaining of dysphonia, difficulty breathing and wheezing for a few hours. He has a history of asthma and has been using his albuterol inhaler without improvement in symptoms. On physical examination the patient is sitting in bed and appears very anxious. High-pitched inspiratory sounds are heard from the side of the bed. Vital signs are HR 114, BP 120/70, RR 28, oxygen saturation 99% on room air, T 37.6℃. No redness or mass is visible on inspection of the oropharynx. No wheezing is heard on lung examination. What is the most likely diagnosis based on the information given? Asthma Epiglottitis Laryngeal mass Paradoxical vocal fold motion

Correct Answer ( D ) Explanation: Paradoxical vocal fold motion is a disorder of respiration that most commonly presents with acute respiratory distress from abnormal adduction of the true vocal folds during inspiration. Patients with this disorder usually present with acute respiratory distress and stridor and are frequently misdiagnosed as anaphylaxis or asthma. When the condition is not recognized, patients may return frequently to the emergency department, leading to delays in diagnosis and potentially unnecessary intensive care unit admissions, intubations and even surgical airway procedures. Physical examination reveals only inspiratory stridor. No other findings, including swelling, redness, masses, expiratory wheeze on lung auscultation or low oxygen saturation are evident. Laryngoscopy can confirm the diagnosis. Treatment includes reassurance and breathing techniques with mild sedatives used as second line therapy. The presentation of paradoxical vocal cord motion is frequently misdiagnosed as asthma (A). However, asthma is typically responsive to bronchodilators and wheezing on expiration is generally heard. One must listen carefully during auscultation to differentiate inspiratory stridor from expiratory wheezing. Epiglottitis (B) is an infectious disorder of the epiglottis that presents with fever, odynophagia, stridor and pooling of secretions. Generally these patients are very still, sitting in a "sniffing position" in the bed and speaking with a muffled voice. This patient's presentation is not consistent with epiglottitis. A laryngeal mass (C) presents with progressive voice changes, hoarseness and inspiratory stridor. Much of the time a mass can be seen or palpated on examina

61-year-old woman fell onto her knees and presents to the Emergency Department with a large left knee effusion. X-rays demonstrate a patella fracture. Urgent orthopedic referral is indicated with which of the following features? Ankle-brachial index of 1.0 Comminuted fracture pattern Positive Lachman test Unable to perform straight leg raise

Correct Answer ( D ) Explanation: Patients who are unable to perform a straight leg raise in the setting of a patella fracture likely have a disruption of the extensor mechanism and need urgent orthopedic referral. Patella fractures may be transverse (most common), comminuted, stellate, vertical, marginal, proximal or distal pole, or osteochondral. They may be displaced or nondisplaced. Transverse fractures are more likely to be widely displaced and involve complete disruption of the extensor mechanism, resulting in the inability to actively extend the knee. This is most commonly the result of forceful quadriceps contraction, forcing the superior patellar fragment superiorly. Standard X-rays, including the anteroposterior, lateral, and sunrise views, are utilized in making the diagnosis. Non-displaced fractures may be managed nonoperatively with long leg cast with the knee in full extension and orthopedic follow up in four to six weeks. An ankle-brachial index of 1.0 (A) is considered within normal limits and does not require specialist consultation. A comminuted fracture pattern (B) is less likely to affect the knee extensor mechanism than displaced transverse fractures of the patella. If the extensor mechanism is intact, consultation is not needed. A positive Lachman test (C) is indicative of an injury to the anterior cruciate ligament. This injury can be managed with outpatient MRI and orthopedic follow up.

Which of the following types of syncope does not increase a patient's risk of death? Cardiac-related syncope Neurologic syncope Syncope of unknown etiology Vasovagal syncope

Correct Answer ( D ) Explanation: Syncope is defined as a transient loss of consciousness associated with loss of postural tone that resolves spontaneously. Following a syncopal event, there should not be any persistent neurologic deficits or altered mental status. The causes of syncope are many and each cause of syncope carries a prognostic risk. Despite complete workups, the cause of syncope remains unknown in nearly 40% of patients. The Framingham Heart Study evaluated the incidence, cause and outcome of syncope among nearly 8000 patients over 17 years. The most frequently identified cause of syncope was vasovagal (21%), followed by cardiac (9.5%), orthostatic (9%), medication related (7%) and neurologic (4%). Patients with vasovagal syncope had no increased risk of death compared to the general population. Cardiac related syncope (A) doubles the risk of death, while neurologic syncope (B) increases the risk of death by 50%. Syncope of unknown etiology (C) also increased the risk of death by 30%.

Which of the following is spared in a LeFort II fracture? Maxilla Nasal bone Orbital wall Zygomatic bone

Correct Answer ( D ) Explanation: The LeFort fractures are midface fractures that involve either complete or partial detachment of the maxilla from the skull. They are classified by their location and the bones involved in the fracture pattern. LeFort I fractures involve a transverse fracture line through the maxilla, detaching the hard palate from the skull. Physical examination shows movement only of the palate and teeth if the palate is moved with the forehead stabilized. A LeFort II fracture is a pyramidal fracture pattern that includes the nasal bridge, maxilla, lacrimal bones, orbital floor, and orbital rim. Physical examination shows movement of the palate and nose when the palate is rocked. A LeFort III fracture, also known as craniofacial dislocation, is characterized by a fracture line that travels transversely through the midface at the level of the nasal bridge and extends through the zygomatic arches. Physical examination shows movement of the entire midface with rocking of the hard palate with the eyes held only in place by the optic nerve. There is increased risk for cerebrovascular involvement in LeFort II and III fractures. LeFort II fractures involve the maxilla (A), nasal bones (B), and orbital wall (C). Involvement of the zygomatic arches distinguishes a LeFort II from a LeFort III fracture.

A 59-year-old woman presents after being the restrained driver in a motor vehicle collision. Vital signs are normal. A CT scan of the chest reveals a nondisplaced sternal fracture with no other intrathoracic injuries. An ECG is normal. She has no increased work of breathing. Which of the following is the best management course? Consult cardiology for a likely blunt cardiac injury Consult orthopedics for operative fixation Perform swallow study with water-soluble contrast to evaluate for occult esophageal injury Provide pain control and discharge

Correct Answer ( D ) Explanation: The patient has a fractured sternum. This injury is usually caused by blunt anterior chest trauma. Sternal fractures are more common in restrained, rather than unrestrained, drivers and passengers in motor vehicle collisions. They are also more common in older rather than younger patients and women more than men. Sternal fractures are associated with rib fractures and lung contusions in greater than 10% of patients. Fractures can be missed on a chest radiograph, and ultrasound may have greater sensitivity than plain radiography. If a sternal fracture is diagnosed on X-ray, a CT scan is recommended to identify other possible intrathoracic injuries. Mediastinal hematomas can be life threatening due to exsanguination and compression of adjacent structures. Treatment relies upon providing pain control. Patients with adequate pain control and no associated injuries can be discharged home. In patients with sternal fractures, only few will develop cardiac complications, such as myocardial contusion. An ECG is recommended. This patient has a normal ECG, so consulting cardiology for a likely blunt cardiac injury (A) is unnecessary. In the absence of significant displacement, most sternal fractures heal well when treated conservatively. Therefore, it is unnecessary to consult orthopedics for operative fixation (B). There is no association between sternal fractures and esophageal rupture. In addition, an esophageal rupture is unlikely in a patient with normal vital signs and a normal CT scan. Therefore, it is unnecessary to perform a swallow study with water-soluble contrast to evaluate for occult esophageal injury (C).

70-year-old man presents to the Emergency Department with sudden onset back pain. He denies any leg weakness or incontinence of his bowel or bladder. His temperature is 99.5°F, blood pressure is 85/40 mm Hg, and his heart rate is 98 bpm. An abdominal ultrasound is performed and the images are shown above. Which of the following is the best management strategy? Gradual return to activity and primary care follow up Initiate esmolol drip and obtain vascular surgery consultation Recommend outpatient, interval imaging Start packed red blood cell transfusion and obtain vascular surgery consultation

Correct Answer ( D ) Explanation: The patient has a ruptured abdominal aortic aneurysm (AAA). This condition is caused by localized stretching and dilation of the aorta. As the aorta increases in size, the risk of rupture increases. An ultrasound of the aorta is performed by obtaining transverse views of the proximal, mid and distal portions of the aorta. The diameter is measured at each point from outer wall to outer wall. A longitudinal view should also be obtained. The normal abdominal aorta is less than 3 cm in diameter. This patient has an aortic diameter greater than 3 cm in diameter and therefore has a AAA. Any patient with back pain and a newly discovered AAA should be treated as if the aorta has ruptured until proven otherwise. Although insensitive for rupture, ultrasound can demonstrate intra-abdominal free fluid. In this case, the ultrasound demonstrates an abdominal aortic aneurysm in the setting of hypotension. Therefore, the best management strategy is to start packed red blood cell transfusion and obtain vascular surgery consultation (D). Gradual return to activity and primary care follow up (A) is appropriate treatment for musculoskeletal back pain without red flags. However, this patient has an emergent surgical condition. Initiation of esmolol drip and obtaining vascular surgery consultation (B) would be the appropriate treatment for an aortic dissection. Esmolol is used to reduce the systolic blood pressure and heart rate in order to decrease the shear stress on the aorta and help reduce the chance of dissection flap propagation. Recommending outpatient, interval imaging (C) is the appropriate management course for asymptomatic, unruptured aneurysms less than 5.5 cm in size.

68-year-old man with a history of long-standing hypertension presents to the emergency department by EMS for acute respiratory distress. His blood pressure is 205/100, heart rate is 130, respiratory rate is 38, and oxygen saturation is 82% on a non-rebreather mask that was applied by the nurse. On thoracic ultrasound of the bilateral anterior chest, you see the image above. Which of the following is the most appropriate next step in management? Administer albuterol 10 mg/hour delivered by continuous nebulization Administer epinephrine 0.3 mg (1:1000) intramuscularly Administer furosemide 20 mg intravenously Begin non-invasive positive pressure ventilation

Correct Answer ( D ) Explanation: The patients severe hypertension and acute respiratory distress along with an ultrasound showing bilateral B-lines is a clinical picture consistent with acute onset cardiogenic pulmonary edema, commonly referred to as flash pulmonary edema. B-lines are an ultrasonographic reverberation artifact that is both sensitive and specific for pulmonary edema (97% and 95%, respectively). In this setting, non-invasive positive pressure ventilation (NIPPV) has been shown to decrease the need for intubation, the number of days spent in the ICU, and mortality in combination with appropriate medical management. NIPPV improves hypoxia by recruiting collapsed alveoli, and helping to force interstitial fluid back into the pulmonary vasculature. NIPPV also increases intrathoracic pressure, which can lead to decreased left ventricular end-diastolic volume. This results in decreased afterload and increased left ventricular ejection fraction. Thus, the heart muscle is stretched less, and placed at a steeper part of the Starling curve. This results in stronger left ventricular contractions, and reductions in blood pressure and heart rate. Furosemide (C), a loop diuretic, is a standard component of medical therapy in the setting of cardiogenic pulmonary edema, but should not be prioritized over respiratory support. Nebulized albuterol (A), a beta-2-agonist, is a crucial component of asthma and COPD treatment, but the clinical presentation combined with the diagnostic accuracy of ultrasound argues strongly against this diagnosis. Epinephrine (B) is the mainstay of treatment for respiratory distress secondary to anaphylaxis.

A 75-year-old man with a history of high blood pressure presents to the ED after a syncopal event. The patient thinks he fell with this episode and is complaining of severe left-sided flank pain. The following bedside abdominal ultrasound is obtained: Which of the following is the next best step in the management plan? Obtain an IV contrast-enhanced CT scan of the abdomen Order a formal radiology department ultrasound Place a Foley catheter to decompress the bladder Vascular surgery consultation

Correct Answer ( D ) Explanation: The ultrasound image shows a large abdominal aortic aneurysm (AAA). The patients history of syncope is concerning for rupture or active leakage. Without rapid surgical intervention mortality approaches 100%. In addition to emergent vascular surgery consultation, the patient should be type and crossmatched for at least four units of blood because large-volume transfusion during definitive repair in the operating room is highly likely.On ultrasound, AAA is measured from outside wall to outside wall at the point of the widest diameter when viewed in cross-section, as seen in the image below. The aorta can also be viewed in long axis to evaluate for luminal irregularities such as thrombus or intimal flap suggesting dissection. However, the long axis should not be used for measurement of aortic diameter because cylinder tangent effect (i.e., visualization of the aorta at a point tangential to the midline) may underestimate the true aortic diameter. Contrast-enhanced CT scan (A) of the abdomen is useful in stable patients with suspected AAA. CT scan is an excellent tool to diagnose leaking or ruptured AAA. Given the ultrasound findings in this patient and a history that is concerning for rupture or leak (syncope and flank pain), the most important next step is to involve a vascular surgeon who can provide definitive care. Formal radiology department ultrasound (B) is a common monitoring technique for stable AAA but is not recommended in the acute setting in an unstable patient. A Foley catheter (C) is used to decompress the bladder in patients with urinary retention. The ultrasound image does not reveal a distended bladder; rather, it is an aneurysmal aorta.

32-year-old man without past medical history presents to the emergency room for knee pain. He was playing football and was tackled, falling onto his knee. He has severe pain, making it difficult to move his knee. An image of his affected knee is shown above. What is the next best step in management? Arthrocentesis Emergent orthopedics consultation Magnetic resonance imaging of the knee Placement in a knee immobilizer and urgent orthopedics follow-up

Correct Answer ( D ) Explanation: This patient has a high-riding patella, which indicates a patellar tendon rupture. These injuries are more common in patients under the age of 40 who have direct trauma to a flexed knee. Risk factors include contact sports, chronic steroid use, and immunosuppression. Diagnosis is made with X-ray imaging showing a high-riding patella along with clinical signs of disruption of the knee extension system. Patients will commonly present with a history of trauma to the knee, immediate pain, difficulty extending the knee, soft tissue swelling, and likely a palpable mass across the inferior quadriceps region (i.e., the patella itself). Patients with partial or full patellar tendon tears should be placed in a knee immobilizer and given urgent orthopedics follow-up as delay in care can lead to poor functional outcomes. This patient has a clear history of trauma and evidence of patellar tendon rupture on imaging, thus an arthrocentesis (A) to rule out infectious or rheumatologic etiologies of his pain is unnecessary. Magnetic resonance imaging (C) is rarely indicated in the emergent setting, especially for joint pain. MRI is reserved for patients with unclear partial thickness injuries or significant intra-articular changes. An emergent orthopedics consultation (B) in the emergency department is not necessary; however, urgent follow-up is indicated as delays in management can lead to quadriceps contractures and adhesions and poor final range of motion outcomes.

17-year-old woman presents to the ED with acute abdominal pain and weakness. Her LMP was approximately six weeks ago. She had a recent positive home pregnancy test but has had no prenatal care to date. The patient has a history of diabetes; her finger stick blood glucose is 377. Vital signs are HR 125, BP 75/54 mm Hg, and RR 16. Other than the abdominal pain, she is sitting up in bed and appears in no distress. A bedside ultrasound is seen above. Which of the following is the most appropriate initial step in her management? Administer insulin and send off an electrolyte panel to evaluate for diabetic ketoacidosis Arrange for an urgent abdominal CT scan Consult the OB/GYN service Place two large-bore IVs and begin fluid resuscitation

Correct Answer ( D ) Explanation: This patient has free fluid in her pelvis on bedside ultrasound and is hypotensive. Although the patient also has elevated blood glucose, a basic tenet of emergency care is to first address the circulation, airway, and breathing. This patient's blood pressure is dangerously low and, in this scenario, ongoing pelvic hemorrhage from a ruptured ectopic pregnancy should be presumed. However, regardless of the cause, the first step is to obtain IV access and begin fluid resuscitation to address her hemodynamic status. It is possible that this patient also has diabetic ketoacidosis (A), given the elevated blood glucose level and her history of diabetes. The initial treatment in DKA is also fluid administration. The administration of insulin will not hurt but can wait until the patient's blood pressure improves. CT scan (B) has no role in the management of suspected ectopic pregnancy (ruptured or not). Even though an abdominal CT scan may be useful in undifferentiated cases, there is enough information in the history and physical examination to suggest that ectopic pregnancy is the definitive diagnosis. The next step is to contact the OB/GYN service (C). This patient needs emergent surgical intervention. In addition, a beta-hCG should be obtained to confirm pregnancy, but this should not delay administration of IV fluids in a hemodynamically unstable patient.

18-year-old college pitcher presents to the ED after being struck in the eye by a baseball. He has severe eye pain and proptosis of the right eye. Visual acuity is 20/25 on the left and hand waving on the right. Which of the following is the most appropriate next step? Administer intravenous acetazolamide Measure intraocular pressure Obtain computed tomography of the orbits Perform lateral canthotomy

Correct Answer ( D ) Explanation: This patient is exhibiting signs and symptoms of a retrobulbar hematoma. A retrobulbar hematoma is an accumulation of blood behind the globe that causes compartment syndrome of the eye via expansion of the hematoma leading to pressure on the optic nerve, globe, and central retinal artery. It is most commonly post-traumatic in etiology. Signs and symptoms include pain, vision loss, an afferent pupillary defect, and proptosis of the globe. If tested, there will be evidence of elevated intraocular pressure. There may also be ophthalmoplegia or paralysis of the extraocular muscles. The diagnosis is largely clinical although a CT of the orbits may be obtained if the diagnosis is unclear. Definitive management is by an emergent lateral canthotomy to relieve rapidly rising orbital pressure which can lead to irreversible vision loss within 90 to 120 minutes of ischemia. Ophthalmology should be consulted immediately as well.

21-year-old woman presents with abdominal pain and vaginal bleeding after a high-speed motor vehicle crash. She states that she is 34 weeks pregnant. On physical exam, you palpate fetal hands and feet with a uterine fundus between the xiphoid process and the umbilicus. Which of the following is the most likely diagnosis? Abruptio placentae Placenta previa Uterine prolapse Uterine rupture

Correct Answer ( D ) Explanation: This patient is presenting with signs and symptoms consistent with uterine rupture. Signs and symptoms of uterine rupture include abdominal pain, sudden termination of uterine contractions, a tearing sensation, vaginal bleeding, fetal heart rate abnormalities, or palpation of fetal parts. Diagnosis is typically clinical although ultrasound may be used if the diagnosis is uncertain. Management of uterine rupture includes rapid delivery by cesarean section and often results in hysterectomy. Oxytocin is contraindicated in these patients as it will often cause worsening of the rupture. Abruptio placentae (A) presents with abdominal pain and vaginal bleeding; however, you should not be able to palpate fetal parts. It is typically diagnosed by ultrasound. Placenta previa (B) presents with painless and often massive vaginal bleeding. This is also diagnosed by ultrasound. Speculum exam should be avoided in these patients as it may worsen the bleeding. Uterine prolapse (C) presents with vaginal bleeding; however, it is usually associated with aggressive traction in an attempt to remove the adherent placenta. It may be associated with the birth itself, but this is more rare.

25-year-old pregnant woman at the end of her third trimester presents to the emergency department with abdominal pain and vaginal bleeding after being kicked by her boyfriend. She has not had any prenatal care and continues to smoke cigarettes. Her blood pressure on arrival is 80/70 mm Hg. What is the next best step in management of this patient's condition? Check coagulation tests Order an obstetric ultrasound Perform a speculum examination Prepare for emergent cesarean section

Correct Answer ( D ) Explanation: This patient presents with severe abruptio placentae, defined as separation of the placenta from the uterine wall which leads to painful vaginal bleeding. This condition is life-threatening for both mother and fetus. Risk factors include trauma, smoking, cocaine use, hypertension, advanced maternal age and multiparity. Diagnosis is made based on clinical suspicion. Patients with signs of shock require volume and blood resuscitation, fetal monitoring and emergent obstetrical consultation for possible emergent cesarean section. Coagulopathy is common, but is often poorly assessed with routine coagulation tests (A) only, and clinical management should be prioritized over laboratory testing. Since the echogenicity of blood and the placenta is similar, ultrasound (B) should not be relied upon to rule out placental abruption. A speculum examination (C) is contraindicated as long as placenta previa has not been ruled out, as it could precipitate severe hemorrhage.

An 85-year-old woman is found unresponsive at her nursing home. EMS arrives on scene, intubates the patient, begins CPR, and administers 3 rounds of epinephrine. The patient arrives in the ED after 20 minutes of "downtime," and you perform a focused ultrasound assessment as seen above. What is the most appropriate next step in management? Administer amiodarone Administer thrombolytics Defibrillate at 200 J Terminate resuscitative efforts

Correct Answer ( D ) Explanation: This video demonstrates a parasternal long axis view of the heart with complete absence of ventricular contractions (cardiac standstill). Cardiac ultrasound is helpful in patients whose pulse cannot be palpated to differentiate between asystole and pulseless electrical activityan important distinction with implications for management and prognosis. In pulseless electrical activity, there is still some cardiac motion and reversible causes may be present. However, patients with cardiac standstill confirmed on ultrasound have little to no chance of survival. Therefore, it is reasonable to terminate resuscitative efforts in this patient. Administration of thrombolytics (B) is reserved for those patients with hemodynamic collapse and suspicion for pulmonary embolism or acute myocardial infarction. Intravenous amiodarone (A) is administered as part of the algorithm to treat ventricular fibrillation and pulseless ventricular tachycardia. It is not part of the asystole algorithm. Defibrillation (C) is also reserved for those patients with ventricular fibrillation and pulseless ventricular tachycardia.

A 52-year-old man presents to the emergency department with the complaint of palpitations. He has a history of type II diabetes mellitus. On physical examination his blood pressure is 180/104 mm Hg, heart rate is 168 bpm and temperature is 102.8°F. You note bilateral exophthalmos and fine tremors in the extremities. An ECG reveals an irregularly irregular rhythm. Propranolol was ordered and administered first. Which of the following should be administered next? Esmolol Hydrocortisone Iodine Propylthiouracil

Correct Answer ( D ) Explanation: Thyroid storm is an acute, life-threatening hypermetabolic state seen in poorly controlled, untreated or unrecognized thyrotoxicosis. Clinical presentation includes fever, tachycardia, hypertension and neurological and gastrointestinal abnormalities. Hypertension is followed by congestive heart failure that may progress to cardiogenic shock. The most common precipitants of thyroid storm are infection and sepsis, but there are a multitude of triggers. The best screening tool for the diagnosis of thyrotoxicosis is thyroid-stimulating hormone (TSH), which is depressed or undetectable in thyrotoxicosis and thyroid storm. Management has a step-wise approach aiming at (1) blocking the sympathetic outflow, (2) blocking the production and release of thyroid hormone, and (3) blocking peripheral conversion of T4 to T3. Propranolol was administered to the patient above decreasing the sympathetic hyperactivity. Next, blocking the production of thyroid hormone can be accomplished by either propylthiouracil (PTU) or methimazole. Both inhibit thyroid peroxidase, therefore inhibiting the binding of iodine to thyroglobulin and blocking production of thyroid hormone. The use of esmolol (A) would aid in blocking the sympathetic outflow, however propranolol was already given and is the drug of choice. Hydrocortisone (B) will inhibit the peripheral conversion of T4 to T3 as well as treat relative adrenal insufficiency commonly seen in thyroid storm. Inhibiting hormone release can be accomplished by giving iodine (C). Iodine blocks the release of thyroid hormone stored in the gland and should be administered one hour after PTU or methimazole.

Which of the following findings on ocular ultrasound is most consistent with a vitreous hemorrhage? Anterior chamber collapse Biconvex mass in the posterior chamber that moves independently of eye movements Echogenic opacity in the most posterior aspect of the posterior chamber Hyperechoic opacities scattered through the vitreou

Correct Answer ( D ) Explanation: Vitreous hemorrhage can occur spontaneously or in the setting of trauma. On ocular ultrasound, it will be seen as a hyperechoic material in the posterior chamber and vitreous. Depending on the extent and age of the bleed, it can have different appearances. Early on, it is characterized by small dots or mobile opacities in the middle portion of the posterior chamber that can be found in a whirlpool-type pattern. Over time, the hemorrhage can become more echogenic and form membranes which can be confused with a retinal detachment. Unlike a retinal detachment, which moves with eye movements, the echogenic membrane associated with vitreous hemorrhage will remain horizontal with eye movement. Distortion of the normal shape of the globe, decreased globe size, buckling of the sclera and anterior chamber collapse (A) can be seen with globe rupture. A lens dislocation is seen as a biconvex mass in the posterior chamber that moves independently of eye movements (B). Retinal detachments are typically seen as an echogenic opacity or membrane in the most posterior aspect of the posterior chamber (D) adjacent to the optic disk, whereas vitreous hemorrhage is typically seen in the midportion of the posterior chamber.

53-year-old woman presents to the Emergency Department with left arm pain after falling onto a wooden coffee table which broke. She has swelling and tenderness of the volar left forearm. Which of the following is initially most appropriate for locating a wooden foreign body in this patient? Computed tomography Magnetic resonance Plain radiography Sonography

Correct Answer ( D ) Explanation: While no single imaging modality can identify all types of foreign bodies, high-resolution sonography has the greatest reliability at identifying wooden soft-tissue foreign bodies. It is also good at identifying foreign bodies composed of plastic, glass, metal, fiber, fish bones, most plant thorns and cactus or sea urchin spines. Foreign bodies will appear hyperechoic with posterior acoustic shadowing behind the foreign object. Different transducer frequencies allow for different tissue penetration and may aid in identifying structures at different depths. For example, a 3.5-MHz transducer may identify an object as much as 10 cm deep, whereas a 12-MHz will identify objects as little as 0.2 to 2.0 cm deep. In addition to localizing foreign bodies, ultrasound may be used to guide local or regional anesthesia or removal. A water bath interface may be required for anatomic locations not amenable to traditional ultrasound gel (e.g., finger web spaces). Computed tomography (A) will identify most wooden foreign bodies. However, it is more expensive than bedside ultrasound, exposes patients to radiation and has a few limitations. For example, wood has a similar radiographic density as water and may be difficult to distinguish from surrounding tissue. The presence of wood or other organic material may also mimic air bubbles. Magnetic resonance (B) is helpful in locating nonmetal radiolucent foreign bodies such as plastic, wood, spines and thorns. It is costly and time-inefficient, so other modalities (e.g., ultrasound, CT) should be utilized prior to this modality. Plain radiography (C) is excellent for identifying metallic foreign bodies, small bones, teeth, certain plastics, gravel and sand. Wood and other organic material is not visible on plain radiographs.


Ensembles d'études connexes

The Phone Call Has Made a Comeback

View Set

Chapter 16: Nursing Management During the Postpartum Period

View Set

Postpartum, Transition to parenthood, Discharge Planning and teaching

View Set

Module 9 TTC (Hormonal/Glucose Regulation B)00

View Set